174
This document consists of 13 printed pages and 1 blank page. © UCLES 2010 [Turn over For Examiner's Use 1 2 3 4 5 6 7 8 9 Total UNIVERSITY OF CAMBRIDGE INTERNATIONAL EXAMINATIONS General Certificate of Education Advanced Level COMPUTING 9691/03 Paper 3 Written For Examination from 2011 SPECIMEN PAPER 2 hours Candidates answer on the Question Paper. No Additional Materials are required. READ THESE INSTRUCTIONS FIRST Write your Centre number, candidate number and name on all the work you hand in. Write in dark blue or black pen. You may use a soft pencil for any diagrams, graphs or rough working. Do not use staples, paper clips, highlighters, glue or correction fluid. Answer all questions. No marks will be awarded for using brand names of software packages or hardware. At the end of the examination, fasten all your work securely together. The number of marks is given in brackets [ ] at the end of each question or part question. M.Mushtaq Hussain Contact the Teacher: 03215275281

UNIVERSITY OF CAMBRIDGE INTERNATIONAL EXAMINATIONS …iteach.pk/wp-content/uploads/2015/02/computer-science-papers-A1-… · COMPUTING 9691/31 Paper 3 May/June 2011 2 hours ... 10

  • Upload
    others

  • View
    0

  • Download
    0

Embed Size (px)

Citation preview

Page 1: UNIVERSITY OF CAMBRIDGE INTERNATIONAL EXAMINATIONS …iteach.pk/wp-content/uploads/2015/02/computer-science-papers-A1-… · COMPUTING 9691/31 Paper 3 May/June 2011 2 hours ... 10

This document consists of 13 printed pages and 1 blank page.

© UCLES 2010 [Turn over

For Examiner's Use

1

2

3

4

5

6

7

8

9

Total

UNIVERSITY OF CAMBRIDGE INTERNATIONAL EXAMINATIONS General Certificate of Education Advanced Level

COMPUTING 9691/03

Paper 3 Written For Examination from 2011

SPECIMEN PAPER

2 hours

Candidates answer on the Question Paper.

No Additional Materials are required.

READ THESE INSTRUCTIONS FIRST

Write your Centre number, candidate number and name on all the work you hand in.

Write in dark blue or black pen.

You may use a soft pencil for any diagrams, graphs or rough working.

Do not use staples, paper clips, highlighters, glue or correction fluid.

Answer all questions.

No marks will be awarded for using brand names of software packages or hardware.

At the end of the examination, fasten all your work securely together.

The number of marks is given in brackets [ ] at the end of each question or part question.

M.Mushtaq Hussain

Contact the Teacher: 03215275281

Page 2: UNIVERSITY OF CAMBRIDGE INTERNATIONAL EXAMINATIONS …iteach.pk/wp-content/uploads/2015/02/computer-science-papers-A1-… · COMPUTING 9691/31 Paper 3 May/June 2011 2 hours ... 10

3

© UCLES 2010 9691/03/SP/11 [Turn over

For

Examiner's

Use

2 (a) A bookshop contains a number of books. Each BOOK is about a single SUBJECT. There may be more than one BOOK about

each SUBJECT. A BOOK may have more than one AUTHOR and each AUTHOR may have written more than one BOOK.

Draw an entity relationship (E–R) diagram to represent this data model in third normal

form and label the relationships. [7] (b) Using examples from this database, explain what is meant by: (i) a primary key,

[2]

(ii) a secondary key,

[2]

(iii) a foreign key.

[2]

M.Mushtaq Hussain

Contact the Teacher: 03215275281

Page 3: UNIVERSITY OF CAMBRIDGE INTERNATIONAL EXAMINATIONS …iteach.pk/wp-content/uploads/2015/02/computer-science-papers-A1-… · COMPUTING 9691/31 Paper 3 May/June 2011 2 hours ... 10

4

© UCLES 2010 9691/03/SP/11

For

Examiner's

Use

3 (a) Explain why an interpreter may be preferred to a compiler as a translator when writing a high-level language program.

[5]

(b) Describe how the code is checked during the syntax analysis stage of compilation.

[3]

4 The contents of the Current Instruction Register (CIR) for one instruction are ADD 01011011 (Note: The ‘ADD’ operation would normally be stored as a binary code.) (i) Explain what is meant by a mnemonic and why it is used.

[3]

M.Mushtaq Hussain

Contact the Teacher: 03215275281

Page 4: UNIVERSITY OF CAMBRIDGE INTERNATIONAL EXAMINATIONS …iteach.pk/wp-content/uploads/2015/02/computer-science-papers-A1-… · COMPUTING 9691/31 Paper 3 May/June 2011 2 hours ... 10

5

© UCLES 2010 9691/03/SP/11 [Turn over

For

Examiner's

Use

(ii) Describe how this address is used if it is an indirect address.

[2]

(iii) Describe how this address is used if it is an indexed address.

[2]

(iv) The address is actually an immediate operand which is to be added to the value

01011101 which is held in the accumulator. Carry out this addition, showing your working.

[3]

M.Mushtaq Hussain

Contact the Teacher: 03215275281

Page 5: UNIVERSITY OF CAMBRIDGE INTERNATIONAL EXAMINATIONS …iteach.pk/wp-content/uploads/2015/02/computer-science-papers-A1-… · COMPUTING 9691/31 Paper 3 May/June 2011 2 hours ... 10

8

© UCLES 2010 9691/03/SP/11

For

Examiner's

Use

6 Describe how memory is managed in a typical computer system. Your answer should include an explanation of: (i) segmentation,

(ii) paging,

(iii) virtual memory.

(You are advised that diagrams may be helpful in your answers.) [9]

M.Mushtaq Hussain

Contact the Teacher: 03215275281

Page 6: UNIVERSITY OF CAMBRIDGE INTERNATIONAL EXAMINATIONS …iteach.pk/wp-content/uploads/2015/02/computer-science-papers-A1-… · COMPUTING 9691/31 Paper 3 May/June 2011 2 hours ... 10

12

© UCLES 2010 9691/03/SP/11

For

Examiner's

Use

9 An import/export company is based in two offices in London and Lahore. Each office has an accounts department and a warehousing department. Each department has a network of computers. It is important that at each office the accounts and warehousing departments must be able to communicate. The London and Lahore offices must also be able to communicate electronically.

With reference to this example, explain the use of the following: (i) copper cabling,

[2]

(ii) wireless communication,

[2]

(iii) routers,

[2]

(iv) bridges,

[2]

M.Mushtaq Hussain

Contact the Teacher: 03215275281

Page 7: UNIVERSITY OF CAMBRIDGE INTERNATIONAL EXAMINATIONS …iteach.pk/wp-content/uploads/2015/02/computer-science-papers-A1-… · COMPUTING 9691/31 Paper 3 May/June 2011 2 hours ... 10

13

© UCLES 2010 9691/03/SP/11

For

Examiner's

Use

(v) modems.

[2]

M.Mushtaq Hussain

Contact the Teacher: 03215275281

Page 8: UNIVERSITY OF CAMBRIDGE INTERNATIONAL EXAMINATIONS …iteach.pk/wp-content/uploads/2015/02/computer-science-papers-A1-… · COMPUTING 9691/31 Paper 3 May/June 2011 2 hours ... 10

This document consists of 13 printed pages and 3 blank pages.

IB11 06_9691_31/2RP © UCLES 2011 [Turn over

*7081239771*

UNIVERSITY OF CAMBRIDGE INTERNATIONAL EXAMINATIONS

General Certificate of Education Advanced Level

COMPUTING 9691/31

Paper 3 May/June 2011

2 hours

Candidates answer on the Question Paper.

No additional materials are required.

No calculators allowed.

READ THESE INSTRUCTIONS FIRST

Write your Centre number, candidate number and name on all the work you hand in.

Write in dark blue or black pen.

You may use a soft pencil for any diagrams, graphs or rough working.

Do not use staples, paper clips, highlighters, glue or correction fluid.

DO NOT WRITE IN ANY BARCODES.

Answer all questions.

No marks will be awarded for using brand names for software packages or hardware.

At the end of the examination, fasten all your work securely together.

The number of marks is given in brackets [ ] at the end of each question or part question.

M.Mushtaq Hussain

Contact the Teacher: 03215275281

Page 9: UNIVERSITY OF CAMBRIDGE INTERNATIONAL EXAMINATIONS …iteach.pk/wp-content/uploads/2015/02/computer-science-papers-A1-… · COMPUTING 9691/31 Paper 3 May/June 2011 2 hours ... 10

2

© UCLES 2011 9691/31/M/J/11

For

Examiner's

Use

1 Name three different types of bus in a processor and state what each is used for.

1

2

3

[6]

2 (a) Explain the relationship between assembly languages and machine code.

[2]

(b) Describe how an assembler produces machine code from assembly language.

[2]

M.Mushtaq Hussain

Contact the Teacher: 03215275281

Page 10: UNIVERSITY OF CAMBRIDGE INTERNATIONAL EXAMINATIONS …iteach.pk/wp-content/uploads/2015/02/computer-science-papers-A1-… · COMPUTING 9691/31 Paper 3 May/June 2011 2 hours ... 10

3

© UCLES 2011 9691/31/M/J/11 [Turn over

For

Examiner's

Use

(c) The address part of a low-level instruction can be the address of the data to be used. This is a direct address.

Describe the following types of addressing: (i) Indirect addressing;

[2]

(ii) Indexed addressing;

[2]

(iii) Relative addressing.

[2]

M.Mushtaq Hussain

Contact the Teacher: 03215275281

Page 11: UNIVERSITY OF CAMBRIDGE INTERNATIONAL EXAMINATIONS …iteach.pk/wp-content/uploads/2015/02/computer-science-papers-A1-… · COMPUTING 9691/31 Paper 3 May/June 2011 2 hours ... 10

4

© UCLES 2011 9691/31/M/J/11

For

Examiner's

Use

3 Discuss the different types of data transmission media. Include a comparison of data transfer rates and transmission ranges in your answer.

[8]

M.Mushtaq Hussain

Contact the Teacher: 03215275281

Page 12: UNIVERSITY OF CAMBRIDGE INTERNATIONAL EXAMINATIONS …iteach.pk/wp-content/uploads/2015/02/computer-science-papers-A1-… · COMPUTING 9691/31 Paper 3 May/June 2011 2 hours ... 10

6

© UCLES 2011 9691/31/M/J/11

For

Examiner's

Use

5 (a) Describe what is meant by the spooling of files.

[2]

(b) (i) State why files which are sent to a shared printer on a local network will be

spooled.

(ii) Explain how this spooling is carried out.

[5]

M.Mushtaq Hussain

Contact the Teacher: 03215275281

Page 13: UNIVERSITY OF CAMBRIDGE INTERNATIONAL EXAMINATIONS …iteach.pk/wp-content/uploads/2015/02/computer-science-papers-A1-… · COMPUTING 9691/31 Paper 3 May/June 2011 2 hours ... 10

7

© UCLES 2011 9691/31/M/J/11 [Turn over

For

Examiner's

Use

6 Describe the purpose of the following parts of a database management system (DBMS). (i) Data Description Language (DDL)

[2]

(ii) Data Manipulation Language (DML)

[3]

7 Part of the information stored in the data dictionary describes the type of data which is

being stored. A particular piece of data is 10010110. State what the data stands for if the data dictionary describes it as: (i) a two's complement binary number;

[1]

(ii) a sign and magnitude binary number;

[1]

(iii) a binary coded decimal number.

[2]

M.Mushtaq Hussain

Contact the Teacher: 03215275281

Page 14: UNIVERSITY OF CAMBRIDGE INTERNATIONAL EXAMINATIONS …iteach.pk/wp-content/uploads/2015/02/computer-science-papers-A1-… · COMPUTING 9691/31 Paper 3 May/June 2011 2 hours ... 10

11

© UCLES 2011 9691/31/M/J/11 [Turn over

For

Examiner's

Use

10 A country has a national football competition based on leagues. Each LEAGUE has a number of TEAMs but each TEAM is only in one LEAGUE. Each TEAM plays at a number of GROUNDs during the season and each GROUND will

host a number of TEAMs during the season. (i) State the relationship between LEAGUE and TEAM.

Draw the entity-relationship (E-R) diagram to show this relationship. [2]

(ii) State the relationship between TEAM and GROUND.

Draw the E-R diagram to show this relationship. [2] (iii) Explain how the relationship between TEAM and GROUND can be designed in

third normal form.

[4]

M.Mushtaq Hussain

Contact the Teacher: 03215275281

Page 15: UNIVERSITY OF CAMBRIDGE INTERNATIONAL EXAMINATIONS …iteach.pk/wp-content/uploads/2015/02/computer-science-papers-A1-… · COMPUTING 9691/31 Paper 3 May/June 2011 2 hours ... 10

12

© UCLES 2011 9691/31/M/J/11

For

Examiner's

Use

11 (a) Describe the differences between interpretation and compilation of a high-level language program.

[3]

(b) When a program is run the processor uses special purpose registers. Describe how the contents of each of the following registers changes during the

fetch-execute cycle: (i) Memory Address Register (MAR)

[2]

(ii) Memory Data Register (MDR)

[2]

M.Mushtaq Hussain

Contact the Teacher: 03215275281

Page 16: UNIVERSITY OF CAMBRIDGE INTERNATIONAL EXAMINATIONS …iteach.pk/wp-content/uploads/2015/02/computer-science-papers-A1-… · COMPUTING 9691/31 Paper 3 May/June 2011 2 hours ... 10

13

© UCLES 2011 9691/31/M/J/11

For

Examiner's

Use

12 A business uses the Internet to communicate with suppliers and to pay bills electronically. Discuss the problems of maintaining confidentiality of data on the Internet and techniques

that can be used to address these problems.

[6]

M.Mushtaq Hussain

Contact the Teacher: 03215275281

Page 17: UNIVERSITY OF CAMBRIDGE INTERNATIONAL EXAMINATIONS …iteach.pk/wp-content/uploads/2015/02/computer-science-papers-A1-… · COMPUTING 9691/31 Paper 3 May/June 2011 2 hours ... 10

UNIVERSITY OF CAMBRIDGE INTERNATIONAL EXAMINATIONS

GCE Advanced Level

MARK SCHEME for the May/June 2011 question paper

for the guidance of teachers

9691 COMPUTING

9691/31 Paper 3 (Written Paper), maximum raw mark 90

This mark scheme is published as an aid to teachers and candidates, to indicate the requirements of the examination. It shows the basis on which Examiners were instructed to award marks. It does not indicate the details of the discussions that took place at an Examiners’ meeting before marking began, which would have considered the acceptability of alternative answers.

Mark schemes must be read in conjunction with the question papers and the report on the examination.

• Cambridge will not enter into discussions or correspondence in connection with these mark schemes. Cambridge is publishing the mark schemes for the May/June 2011 question papers for most IGCSE, GCE Advanced Level and Advanced Subsidiary Level syllabuses and some Ordinary Level syllabuses.

M.Mushtaq Hussain

Contact the Teacher: 03215275281

Page 18: UNIVERSITY OF CAMBRIDGE INTERNATIONAL EXAMINATIONS …iteach.pk/wp-content/uploads/2015/02/computer-science-papers-A1-… · COMPUTING 9691/31 Paper 3 May/June 2011 2 hours ... 10

Page 2 Mark Scheme: Teachers’ version Syllabus Paper

GCE A LEVEL – May/June 2011 9691 31

© University of Cambridge International Examinations 2011

1 e.g.-Data bus -to carry data from one location to another in processor // e.g. from MDR to CIR -Address bus -carries the address of a memory location // e.g. Address of location in memory from MAR -Control bus -Carries control signals around processor // to synchronise the operation of the processor components // by example: memory read/write completed // each line carries a different signal. Accept: system bus, memory bus, firewire, USB, PCI + explanation (2nd mark is dependent on correct bus name) (2 per -, max 6) [6]

2 (a) -One to one

-Mnemonics are used to represent operation codes -Labels are used to represent memory addresses -machine code is binary codes (only) -assembly code can not be executed // machine code can be executed -machine code and assembly language are both low level languages (machine specific) (1 per -, max 2) [2]

(b) -Labels added to a symbol table

-Labels are later looked up to determine the actual address / Assembler must allocate addresses to labels -Mnemonic looked up in opcode table to find operation code -Macro instructions used to stand for groups of instructions (1 per -, max 2) [2]

(c) (i) -Address in instruction is the address of the address of / pointer to the location...

-which contains the data to be used [2] (ii) -Address in the instruction has added to it

-the contents of the Index Register/IR [2] (iii) -Address in the Instruction is the displacement

-from the address of the first/current instruction -the value is added to the PC [2]

M.Mushtaq Hussain

Contact the Teacher: 03215275281

Page 19: UNIVERSITY OF CAMBRIDGE INTERNATIONAL EXAMINATIONS …iteach.pk/wp-content/uploads/2015/02/computer-science-papers-A1-… · COMPUTING 9691/31 Paper 3 May/June 2011 2 hours ... 10

Page 3 Mark Scheme: Teachers’ version Syllabus Paper

GCE A LEVEL – May/June 2011 9691 31

© University of Cambridge International Examinations 2011

3 -Coaxial cable -description/one transmission medium (copper) surrounded by insulation -Twisted pair -description/two conducting wires twisted around each other -Optic fibre -many fibres contained -description/fine glass strands carry light signals // optic fibre is very fragile -Interference free -Wireless communication -Radio signals -open to interception / latency / uses WEP keys for security -Infrared/Microwave -restricted by line of sight -transfer rate statement -range statement (1 per -, max 8) [8]

4 (a) -A system in which the output is produced quickly enough to affect the next input /current

process -a system that reacts fast enough to influence behaviour in the outside world [1]

(b) -A number of sensors stationed around apartment

-Temperature/humidity sensor sends temperature/humidity to processor -Use analogue to digital converter to convert the temperature/humidity measurements -Processor decides whether air conditioning system is in operation -(processor) compares measured temperature/humidity to required temperature/humidity -If necessary actuator is used to adjust settings/turn on cooling/heating/humidifier -Delay before next reading is taken from temperature/humidity sensor. // temperature readings are sampled // taking readings is repeated -Sensors on windows to warn if they are open during operation. (1 per -, max 4) [4]

(c) Any suitable real-time or pseudo-real-time application e.g.

-To play a racing game -So that the player can steer the car realistically -any reservation type system -to prevent double booking [2]

M.Mushtaq Hussain

Contact the Teacher: 03215275281

Page 20: UNIVERSITY OF CAMBRIDGE INTERNATIONAL EXAMINATIONS …iteach.pk/wp-content/uploads/2015/02/computer-science-papers-A1-… · COMPUTING 9691/31 Paper 3 May/June 2011 2 hours ... 10

Page 4 Mark Scheme: Teachers’ version Syllabus Paper

GCE A LEVEL – May/June 2011 9691 31

© University of Cambridge International Examinations 2011

5 (a) -Temporarily storing data for output later -several computers can send data to be printed at the same time -when queuing jobs sent to a single device (1 per -, max 2) [2]

(b) (i) -Jobs can be queued to ensure that none are missed

-Stops jobs being frozen/lost when printer unavailable -complete documents are printed

(ii) -print jobs are stored on secondary storage

-jobs can be given a print priority -jobs are maintained by a queue / priority queue data structure -data structure consists of reference data to each print job -When printer free, job with highest priority / at head of queue is printed -print files are sent from secondary storage to print buffer. (1 per -, max 4 per dotty, max 5) [5]

6 (i) -language to describe/alter table designs (NOT file)

-includes Identifiers/data type/relationships -any validation rules that the data must adhere to… (1 per -, max 2) [2]

(ii) -designed to allow a user to query/retrieve data/sort the database

-insert / delete / update -data in the database / table(s) (1 per -, max 3) [3]

7 (i) –106 [1] (ii) –22 [1] (iii) 96 (1 per digit) [2]

M.Mushtaq Hussain

Contact the Teacher: 03215275281

Page 21: UNIVERSITY OF CAMBRIDGE INTERNATIONAL EXAMINATIONS …iteach.pk/wp-content/uploads/2015/02/computer-science-papers-A1-… · COMPUTING 9691/31 Paper 3 May/June 2011 2 hours ... 10

Page 5 Mark Scheme: Teachers’ version Syllabus Paper

GCE A LEVEL – May/June 2011 9691 31

© University of Cambridge International Examinations 2011

8 (a) (i) -A dynamic data structure changes size // A static data structure has the same size -dynamic data structure matches size to data requirements // static data structure takes no account of data requirements -dynamic data structure takes memory from heap as required -static data structure is predefined at compile time [2]

(ii) Advantage:

-Array is of fixed size which simplifies algorithms // or by example e.g. retrieval of data -Array controls the maximum size of the queue Disadvantage: -Queue held in an array cannot expand beyond the size of the array -If queue is small then memory space is wasted. [2]

(b) (i) Either:

Start FreeSpace

Start: 5 1 FRO 4 Start: 4 0 FRO 3

NFree: 6 2 TSI X NFree: 5 1 TSI X

3 DON 1 2 DON 0

4 ROS 2 3 ROS 1

5 BEV 3 4 BEV 2

6 5

Mark as follows ... -Start pointer + some value/arrows -All values included -Null pointer -Indication of free space Diagram in arrival order -in arrival order -with correct pointers OR Diagram in alphabetical order -with correct pointers OR Array diagram -in arrival order -correct pointers (1 per -, max 5) [5]

BEV DON FRO ROS TSI

M.Mushtaq Hussain

Contact the Teacher: 03215275281

Page 22: UNIVERSITY OF CAMBRIDGE INTERNATIONAL EXAMINATIONS …iteach.pk/wp-content/uploads/2015/02/computer-science-papers-A1-… · COMPUTING 9691/31 Paper 3 May/June 2011 2 hours ... 10

Page 6 Mark Scheme: Teachers’ version Syllabus Paper

GCE A LEVEL – May/June 2011 9691 31

© University of Cambridge International Examinations 2011

(ii) -Input NewItem -Store NewItem in next free space -Set Current to value at Start -Read values in list following pointers. -until Current value in list > NewItem -Pointer of Previous points to NewItem -NewItem points to Current -update free space list -Mention of any special cases e.g. NewItem being First in list // list empty // list full // no free space (1 per -, max 5) [5]

9 (a) -reverse Polish expressions can be processed directly from left to right

-Is free of ambiguities -does not require brackets -does not require use of rules of precedence (1 per -, max 2) [2]

+ _ * _ (b) e d d-e b c c c*(d-e) a a+b a+b a+b a+b (a+b)-c*(d-e)

Mark points: -at least two operators shown between transitions -a and b in first stage -a+b after first operator -e,d,c, (a+b) in stack in correct order -(d-e) -c*(d-e) -(a+b)-c*(d-e) (1 per -, max 6) [6]

10 (i) One to many [2] (ii) Many to many [2] (iii) -Link table needed...

-with primary key made up of combination of primary keys of TEAM and GROUND -Primary keys of TEAM and GROUND used as foreign keys in link table -This turns the many to many relationship into..// a many-to-many relationship can not be implemented -One-to-many and many-to-one/ 2x one-to-many relationships (1 per -, max 4) [4]

LEAGUE TEAM

TEAM GROUND

M.Mushtaq Hussain

Contact the Teacher: 03215275281

Page 23: UNIVERSITY OF CAMBRIDGE INTERNATIONAL EXAMINATIONS …iteach.pk/wp-content/uploads/2015/02/computer-science-papers-A1-… · COMPUTING 9691/31 Paper 3 May/June 2011 2 hours ... 10

Page 7 Mark Scheme: Teachers’ version Syllabus Paper

GCE A LEVEL – May/June 2011 9691 31

© University of Cambridge International Examinations 2011

11 (a) -Interpreter translates one instruction, runs it before going on to the next // Compiler translates all the instructions before run. -Compiler creates object code/executable file // Interpreter does not -Interpreter makes for easier debugging -Compiled programs will execute faster // interpreted code will execute slower -Interpreter must be present to run the program // compiler not needed at runtime -Interpreter will translate code in loops more than once // Compiler only once -once compiled no further translation needed // every program execution requires interpreter (1 per -, max 3) [3]

(b) (i) -Contents copied from PC -Contents changed to the operand/address part of CIR [2] (ii) -Instruction copied from memory/location to MDR when contents of MAR are from PC -Data copied from memory/location to MDR when instruction is LOAD -Data copied from ALU/Accumulator to MDR when instruction is STORE [max 2] 12 -Must safeguard against unauthorised access to the computer system

-Firewall used to restrict access to known sources -Control access to the network using accounts/user IDs with passwords // procedures in place for authentication -File contents can be encrypted -procedures in place to protect against malware -all payments/communication can be made through a secure connection -need to safeguard against bogus websites -Procedures in place for authorisation of resources -Users allocated access rights to various resources // users have access to certain files/folders only -Files can be password protected / read-only -users can access the network from certain terminals only / certain times of the day only -use of digital signatures (1 per -, max 6) [6]

M.Mushtaq Hussain

Contact the Teacher: 03215275281

Page 24: UNIVERSITY OF CAMBRIDGE INTERNATIONAL EXAMINATIONS …iteach.pk/wp-content/uploads/2015/02/computer-science-papers-A1-… · COMPUTING 9691/31 Paper 3 May/June 2011 2 hours ... 10

This document consists of 12 printed pages.

IB11 11_9691_31/2RP © UCLES 2011 [Turn over

*2788786869*

UNIVERSITY OF CAMBRIDGE INTERNATIONAL EXAMINATIONS

General Certificate of Education Advanced Level

COMPUTING 9691/31

Paper 3 October/November 2011

2 hours

Candidates answer on the Question Paper.

No additional materials are required.

No calculators allowed.

READ THESE INSTRUCTIONS FIRST

Write your Centre number, candidate number and name on all the work you hand in.

Write in dark blue or black pen.

You may use a soft pencil for any diagrams, graphs or rough working.

Do not use staples, paper clips, highlighters, glue or correction fluid.

DO NOT WRITE IN ANY BARCODES.

Answer all questions.

No marks will be awarded for using brand names for software packages or hardware.

At the end of the examination, fasten all your work securely together.

The number of marks is given in brackets [ ] at the end of each question or part question.

M.Mushtaq Hussain

Contact the Teacher: 03215275281

Page 25: UNIVERSITY OF CAMBRIDGE INTERNATIONAL EXAMINATIONS …iteach.pk/wp-content/uploads/2015/02/computer-science-papers-A1-… · COMPUTING 9691/31 Paper 3 May/June 2011 2 hours ... 10

2

© UCLES 2011 9691/31/O/N/11

For

Examiner's

Use

1 Describe the following components of a typical PC operating system and explain how they are used.

(a) File allocation table (FAT)

[3]

(b) Boot file

[3]

M.Mushtaq Hussain

Contact the Teacher: 03215275281

Page 26: UNIVERSITY OF CAMBRIDGE INTERNATIONAL EXAMINATIONS …iteach.pk/wp-content/uploads/2015/02/computer-science-papers-A1-… · COMPUTING 9691/31 Paper 3 May/June 2011 2 hours ... 10

3

© UCLES 2011 9691/31/O/N/11 [Turn over

For

Examiner's

Use

2 (a) Explain what is meant by Von Neumann architecture.

[3]

(b) Describe the use of the following special purpose registers and how they change

during the fetch-execute cycle. (i) Program Counter (PC)

[3]

(ii) Current Instruction Register (CIR)

[3]

M.Mushtaq Hussain

Contact the Teacher: 03215275281

Page 27: UNIVERSITY OF CAMBRIDGE INTERNATIONAL EXAMINATIONS …iteach.pk/wp-content/uploads/2015/02/computer-science-papers-A1-… · COMPUTING 9691/31 Paper 3 May/June 2011 2 hours ... 10

4

© UCLES 2011 9691/31/O/N/11

For

Examiner's

Use

3 (a) Convert the following denary numbers into 10-bit, sign and magnitude, binary numbers: (i) -390

(ii) -47

[3]

(b) Convert the following denary numbers into 8-bit, two's complement, binary numbers: (i) +93

[2]

(ii) - 69

[2]

M.Mushtaq Hussain

Contact the Teacher: 03215275281

Page 28: UNIVERSITY OF CAMBRIDGE INTERNATIONAL EXAMINATIONS …iteach.pk/wp-content/uploads/2015/02/computer-science-papers-A1-… · COMPUTING 9691/31 Paper 3 May/June 2011 2 hours ... 10

5

© UCLES 2011 9691/31/O/N/11 [Turn over

For

Examiner's

Use

(c) (i) Using the binary values from part (a) work out (-390) + (- 47), giving your answer in sign and magnitude form using 10-bit binary. You must show your working.

[3]

(ii) Using the binary values from part (b), work out 93 - 69, giving your answer in two's

complement form using 8-bit binary. You must show your working.

[4]

M.Mushtaq Hussain

Contact the Teacher: 03215275281

Page 29: UNIVERSITY OF CAMBRIDGE INTERNATIONAL EXAMINATIONS …iteach.pk/wp-content/uploads/2015/02/computer-science-papers-A1-… · COMPUTING 9691/31 Paper 3 May/June 2011 2 hours ... 10

6

© UCLES 2011 9691/31/O/N/11

For

Examiner's

Use

4 A health ministry has decided that it would be useful for doctors in that country to communicate using an intranet.

Patient records could be shared and advice could be given by the doctors. (a) Describe what is meant by an intranet.

[3]

(b) Explain why an intranet was used rather than an open network like the World Wide

Web.

[5]

M.Mushtaq Hussain

Contact the Teacher: 03215275281

Page 30: UNIVERSITY OF CAMBRIDGE INTERNATIONAL EXAMINATIONS …iteach.pk/wp-content/uploads/2015/02/computer-science-papers-A1-… · COMPUTING 9691/31 Paper 3 May/June 2011 2 hours ... 10

9

© UCLES 2011 9691/31/O/N/11 [Turn over

For

Examiner's

Use

7 (a) Explain the difference between storing data in a flat file and in a relational database.

[2]

(b) Data about patients, doctors and treatments in a hospital are stored in a relational

database. Explain the advantages of using a relational database rather than a flat file to store the

hospital data.

[3]

(c) (i) Explain why access to the data in the database needs to be controlled.

(ii) Describe how this can be achieved.

[5]

M.Mushtaq Hussain

Contact the Teacher: 03215275281

Page 31: UNIVERSITY OF CAMBRIDGE INTERNATIONAL EXAMINATIONS …iteach.pk/wp-content/uploads/2015/02/computer-science-papers-A1-… · COMPUTING 9691/31 Paper 3 May/June 2011 2 hours ... 10

11

© UCLES 2011 9691/31/O/N/11 [Turn over

For

Examiner's

Use

9 (a) (i) Describe what happens during the lexical analysis phase of compilation.

[4]

(ii) Explain how syntax errors are identified during compilation.

[3]

(b) (i) Explain the value of using library routines when writing new programs.

[2]

(ii) Describe how linkers and loaders are used to make the use of library routines

possible.

[2]

M.Mushtaq Hussain

Contact the Teacher: 03215275281

Page 32: UNIVERSITY OF CAMBRIDGE INTERNATIONAL EXAMINATIONS …iteach.pk/wp-content/uploads/2015/02/computer-science-papers-A1-… · COMPUTING 9691/31 Paper 3 May/June 2011 2 hours ... 10

Page 2 Mark Scheme: Teachers’ version Syllabus Paper

GCE A LEVEL – October/November 2011 9691 31

© University of Cambridge International Examinations 2011

1 (a) -disk space is organised into allocation units (clusters) -FAT is a map of which clusters are used to store which files // Individual sectors on the disk

are organised into clusters which are used to store the files -Details where files are stored on backing store -Acts as an index on the hard drive -Shows unused/unusable clusters (1 per -, max 3) [3] (b) -The boot file contains user-defined information... -to tailor the operating system // Contains parameters by which the system will operate -boot file stored on backing store/CMOSRAM... -read/written to by the boot program (held on ROM) (1 per -, max 3) [3] 2 (a) -Single processor/control unit -Sequential processing of program instructions -Instructions and data indistinguishable -Can be stored together in same memory unit -programs can be exchanged/reloaded easily to the same memory unit (1 per -, max 3) [3] (b) (i) -Contains the address of the next instruction to be fetched -Passes address to the MAR -and is then incremented -Contents altered to the operand of the instruction is a jump instruction (1 per -, max 3) [3] (ii) -Holds the current instruction... -divided into the op-code and operand -Holds the instruction while the op-code is decoded -Sends the address to the MAR. -Mark for mention of use of address to alter PC/need for other parts of instruction e.g.

addressing type used. (1 per -, max 3) [3]

M.Mushtaq Hussain

Contact the Teacher: 03215275281

Page 33: UNIVERSITY OF CAMBRIDGE INTERNATIONAL EXAMINATIONS …iteach.pk/wp-content/uploads/2015/02/computer-science-papers-A1-… · COMPUTING 9691/31 Paper 3 May/June 2011 2 hours ... 10

Page 3 Mark Scheme: Teachers’ version Syllabus Paper

GCE A LEVEL – October/November 2011 9691 31

© University of Cambridge International Examinations 2011

3 (a) (i) –390 = 1110000110 (ii) –47 = 1000101111 (1 for both sign bits and 1 each for the magnitude parts) [3] (b) (i) +93 = 01011101 [2] (ii) -69 = 10111011 [2] (in each case 1 mark per nybble) (c) (i) 1 1 1 0 0 0 0 1 1 0 1 0 0 0 1 0 1 1 1 1 1 1 1 0 1 1 0 1 0 1 1 1 1 =1 1 1 0 1 1 0 1 0 1 (1 for correct carries, 1 for correct answer (9 bits), 1 for correct msb) [3] f.t. (ii) 0 1 0 1 1 1 0 1 + 1 0 1 1 1 0 1 1 = 1 0 0 0 1 1 0 0 0 1 1 1 1 1 1 1 1 (1 for correct sum, 1 for correct answer (8 bits), 1 for correct carries, 1 for showing

ignoring ninth bit) [4] 4 (a) -networked communication system...// content provided by a web server -probably provided on the Internet -Restricted access... -to specific members authorised by the health ministry -Access is password controlled -Content is webpages / made available from a web server / viewed using browser software (1 per -, max 3) [3] (b) e.g.-Limited number of users speeds up access... -Information being communicated is sensitive/confidential... -needs protection from being seen by unauthorised people -Information on system will be relevant/easily updated -Less information makes it easier to navigate -easier to control who can access the content (1 per -, max 5) [5]

M.Mushtaq Hussain

Contact the Teacher: 03215275281

Page 34: UNIVERSITY OF CAMBRIDGE INTERNATIONAL EXAMINATIONS …iteach.pk/wp-content/uploads/2015/02/computer-science-papers-A1-… · COMPUTING 9691/31 Paper 3 May/June 2011 2 hours ... 10

Page 4 Mark Scheme: Teachers’ version Syllabus Paper

GCE A LEVEL – October/November 2011 9691 31

© University of Cambridge International Examinations 2011

5 (a) E.g. -Touch sensor/pressure sensor/infrared sensor / other sensible -Needed to tell robot when components arrive // To investigate orientation of component/ or

anything sensible E.g. -Actuator (electric motor/stepper motor/end effecter) of some sort -Needed to move robot arm // to physically interact with component // or anything sensible -(Speaker/LCD display) conditional on: -a description of error reporting (2 or 0 marks) (1 per -, max 4) [4] (b) e.g. -Cheaper, do not need to be paid -Work 24/7 -Do not require heat, light, space, ventilation, facilities -robots can work in hazardous environments -Items/actions produced are all to a consistent high standard // fewer errors -Reliable/workers can be off work/will never strike -Actions are more accurate than those of human. (1 per -, max 4) [4] (c) -May involve simply changing from one stored program to another -set new parameters for current program -edit program/writing new program code -by physically being moved through intermediate positions … -…which the system can then replicate (1 per -, max 3) [3] 6 -Interrupts have a priority -on receipt of an interrupt all interrupts of a lower priority are “masked out”/refused -Contents of registers are placed on stack -the appropriate interrupt service routine (ISR) is loaded and run -on completion of the ISR … - …values read from stack and loaded to registers -all interrupts are re-enabled -the interrupted process is resumed -Mention of use of vectored interrupt to point to code that needs to be run. (1 per -, max 6) [6]

M.Mushtaq Hussain

Contact the Teacher: 03215275281

Page 35: UNIVERSITY OF CAMBRIDGE INTERNATIONAL EXAMINATIONS …iteach.pk/wp-content/uploads/2015/02/computer-science-papers-A1-… · COMPUTING 9691/31 Paper 3 May/June 2011 2 hours ... 10

Page 5 Mark Scheme: Teachers’ version Syllabus Paper

GCE A LEVEL – October/November 2011 9691 31

© University of Cambridge International Examinations 2011

7 (a) -A flat file is a datafile with records of the same structure -A relational database consists of a number of tables that are linked -flat file may contain redundant/duplicated data [2] (b) -Reduces repetition/duplication of data items // keeps physical volume of data to a minimum

// minimises redundant data -Increases data integrity // reduces data inconsistency -(Simpler) data retrieval through queries // reports are easy to generate -Amending/searching/sorting data is easier -Amending the data structures (e.g. to include wards) is simpler to implement -Changes to the data structure will not affect existing applications programs // Program/data

independence (1 per -, max 3) [3] (c) (i),(ii) -Information on patients is sensitive -certain data needs to be restricted to certain users -avoid concurrent updates of the same record -Patient table can be encrypted -groups of users can be set up with the same privileges/access rights -…Which are arranged in a hierarchy -access rights dictate what the user is allowed to see/do -Access rights relate to a table/ tables / fields/attributes /queries/reports… -access rights may determine the HCI provided to the user. (1 per -, max 5) [5] 8 (a) (i) -Variable whose scope is restricted to a specific procedure/function/module/block (ii) -Variable whose scope exists across an entire program (iii) -procedure call passes the value of the parameter -local copy of data is used // discarded when procedure finished (iv) -pointer/address of the parameter/variable is passed -any changes are retained after returning to the calling code. [4] (b) -Return address is stored on stack (before control passed to procedure) -Values of parameters are placed on stack -Procedure will read same number of values from stack as parameters it was expecting -Address can be read from stack and placed in PC -Nested calls to procedures will be unwound in correct order -Problems if too many nested calls (unending recursive function) // stack overflow (1 per -, max 4) [4]

M.Mushtaq Hussain

Contact the Teacher: 03215275281

Page 36: UNIVERSITY OF CAMBRIDGE INTERNATIONAL EXAMINATIONS …iteach.pk/wp-content/uploads/2015/02/computer-science-papers-A1-… · COMPUTING 9691/31 Paper 3 May/June 2011 2 hours ... 10

Page 6 Mark Scheme: Teachers’ version Syllabus Paper

GCE A LEVEL – October/November 2011 9691 31

© University of Cambridge International Examinations 2011

9 (a) (i) -Strings of characters are grouped to form keywords/reserved words -Checks reserved words for validity -keywords/Reserved words/identifiers replaced by tokens -identifiers placed in symbol table -Unnecessary characters/comments/whitespace removed -final output is a token string (1 per -, max 4) [4] (ii) -the format of instruction/token string is compared … -… to forms for acceptable expressions and statements. - as defined by the meta language used -example of a syntax error e.g. IF THEN x=3 (1 per -, max 3) [3] (b) (i) Routines are: -Already written and can be inserted with a single command word/Saves time in writing

code -many projects require similar code e.g. sorting/searching -Already tested -Code is robust and reliable -should ensure consistency of standards -Already translated/Makes translation process faster/simpler (1 per -, max 2) [2] (ii) -Loader is used to load routine into memory when required -Ensures no memory conflicts between different routines -Linker links segments/files of code (to produce executable code). [2] 10 (a) (i) There can only be one <non-zero-digit> before the letter <group> (ii) X is not defined as a <letter> (iii) Only one <digit> is allowed after <group> [3] (b) Mark Points: -Only one entry and one exit point used -Order correct (non-zero-digit, letter, digit) -Loop around letter to create group -Loop around digit to create two options one to include final digit, one to omit final digit (1 per -, max 4) [4]

letter non-zero-digit digit

variable-identifier

M.Mushtaq Hussain

Contact the Teacher: 03215275281

Page 37: UNIVERSITY OF CAMBRIDGE INTERNATIONAL EXAMINATIONS …iteach.pk/wp-content/uploads/2015/02/computer-science-papers-A1-… · COMPUTING 9691/31 Paper 3 May/June 2011 2 hours ... 10

This document consists of 12 printed pages.

IB11 11_9691_33/4RP © UCLES 2011 [Turn over

*7117681918*

UNIVERSITY OF CAMBRIDGE INTERNATIONAL EXAMINATIONS

General Certificate of Education Advanced Level

COMPUTING 9691/33

Paper 3 October/November 2011

2 hours

Candidates answer on the Question Paper.

No additional materials are required.

No calculators allowed.

READ THESE INSTRUCTIONS FIRST

Write your Centre number, candidate number and name on all the work you hand in.

Write in dark blue or black pen.

You may use a soft pencil for any diagrams, graphs or rough working.

Do not use staples, paper clips, highlighters, glue or correction fluid.

DO NOT WRITE IN ANY BARCODES.

Answer all questions.

No marks will be awarded for using brand names for software packages or hardware.

At the end of the examination, fasten all your work securely together.

The number of marks is given in brackets [ ] at the end of each question or part question.

M.Mushtaq Hussain

Contact the Teacher: 03215275281

Page 38: UNIVERSITY OF CAMBRIDGE INTERNATIONAL EXAMINATIONS …iteach.pk/wp-content/uploads/2015/02/computer-science-papers-A1-… · COMPUTING 9691/31 Paper 3 May/June 2011 2 hours ... 10

2

© UCLES 2011 9691/33/O/N/11

For

Examiner's

Use

1 (a) State what is meant by the boot (bootstrap) program.

[2]

(b) Explain how the boot program is used when a PC is turned on.

[4]

M.Mushtaq Hussain

Contact the Teacher: 03215275281

Page 39: UNIVERSITY OF CAMBRIDGE INTERNATIONAL EXAMINATIONS …iteach.pk/wp-content/uploads/2015/02/computer-science-papers-A1-… · COMPUTING 9691/31 Paper 3 May/June 2011 2 hours ... 10

3

© UCLES 2011 9691/33/O/N/11 [Turn over

For

Examiner's

Use

2 (a) Describe the use of the Memory Data Register (MDR). Explain how the contents change during the fetch-execute cycle.

[3]

(b) Name three types of bus that are used in a computer. For each one explain what it is

used for.

[6]

M.Mushtaq Hussain

Contact the Teacher: 03215275281

Page 40: UNIVERSITY OF CAMBRIDGE INTERNATIONAL EXAMINATIONS …iteach.pk/wp-content/uploads/2015/02/computer-science-papers-A1-… · COMPUTING 9691/31 Paper 3 May/June 2011 2 hours ... 10

4

© UCLES 2011 9691/33/O/N/11

For

Examiner's

Use

3 (a) Convert the following denary numbers into 8-bit, sign and magnitude, binary numbers: (i) +39

(ii) - 47

[3]

(b) Convert the following denary numbers into 8-bit, two's complement, binary numbers: (i) - 3

[2]

(ii) - 47

[2]

M.Mushtaq Hussain

Contact the Teacher: 03215275281

Page 41: UNIVERSITY OF CAMBRIDGE INTERNATIONAL EXAMINATIONS …iteach.pk/wp-content/uploads/2015/02/computer-science-papers-A1-… · COMPUTING 9691/31 Paper 3 May/June 2011 2 hours ... 10

5

© UCLES 2011 9691/33/O/N/11 [Turn over

For

Examiner's

Use

(c) A particular computer uses a single 10-bit word to store a floating-point representation of a number.

The first 6 bits are used to store the mantissa and the remaining 4 bits are used to store the exponent.

(i) Explain why 000101 0100 = 21

2 using this notation.

[2]

(ii) Rewrite the binary value of this floating-point representation so that it is in

normalised form.

[2]

(iii) 011001 0011 is a normalised floating-point number. By converting each of the mantissa and the exponent into a denary number first,

write this number in denary.

[3]

M.Mushtaq Hussain

Contact the Teacher: 03215275281

Page 42: UNIVERSITY OF CAMBRIDGE INTERNATIONAL EXAMINATIONS …iteach.pk/wp-content/uploads/2015/02/computer-science-papers-A1-… · COMPUTING 9691/31 Paper 3 May/June 2011 2 hours ... 10

6

© UCLES 2011 9691/33/O/N/11

For

Examiner's

Use

4 The health system in a country consists of a number of local surgeries. Individual doctors give medical assistance to people in the immediate area. In addition to this there are a number of main hospitals. Each hospital provides services which the doctors are unable to provide in their surgeries.

The main hospitals and the surgeries which they serve are linked by using an intranet. (a) Describe what is meant by an intranet.

[3]

(b) Explain the advantages and disadvantages of using an intranet rather than the Internet.

[5]

M.Mushtaq Hussain

Contact the Teacher: 03215275281

Page 43: UNIVERSITY OF CAMBRIDGE INTERNATIONAL EXAMINATIONS …iteach.pk/wp-content/uploads/2015/02/computer-science-papers-A1-… · COMPUTING 9691/31 Paper 3 May/June 2011 2 hours ... 10

8

© UCLES 2011 9691/33/O/N/11

For

Examiner's

Use

6 (a) Describe the advantages of using a relational database to store data rather than a flat file.

[3]

(b) Data about guests in a hotel are stored in a relational database. One part of the database consists of a table of GUEST details. This is linked to a table ACCOUNT. Whenever a guest orders something in the hotel, the charge for that service is stored in the ACCOUNT table.

(i) State the primary key of the GUEST table, justifying your choice.

[2]

(ii) State a secondary key in the ACCOUNT table, justifying your choice.

[2]

(iii) State what is meant by a foreign key.

[1]

(iv) State a foreign key in the ACCOUNT table, justifying your choice.

[2]

M.Mushtaq Hussain

Contact the Teacher: 03215275281

Page 44: UNIVERSITY OF CAMBRIDGE INTERNATIONAL EXAMINATIONS …iteach.pk/wp-content/uploads/2015/02/computer-science-papers-A1-… · COMPUTING 9691/31 Paper 3 May/June 2011 2 hours ... 10

9

© UCLES 2011 9691/33/O/N/11 [Turn over

For

Examiner's

Use

7 (a) Explain how variables are managed during the different stages of compilation of a high-level language program.

[7]

(b) (i) Give two advantages of using a compiler rather than an interpreter to translate a

high-level language program.

[2]

(ii) Describe an advantage of using an interpreter rather than a compiler to translate a

high-level language program.

[2]

M.Mushtaq Hussain

Contact the Teacher: 03215275281

Page 45: UNIVERSITY OF CAMBRIDGE INTERNATIONAL EXAMINATIONS …iteach.pk/wp-content/uploads/2015/02/computer-science-papers-A1-… · COMPUTING 9691/31 Paper 3 May/June 2011 2 hours ... 10

Page 2 Mark Scheme: Teachers’ version Syllabus Paper

GCE A LEVEL – October/November 2011 9691 33

© University of Cambridge International Examinations 2011

1 (a) -A short sequence of machine code instructions -Stored in the ROM -available when the power is switched on -Used to load the operating system (1 per -, max 2) [2] (b) -Power-up process places address of first instruction in PC -Carries out the POST (power on self-test) -Uses the user-defined parameters in the boot file to configure the system -Mention of BIOS/autoexec.bat/config.sys… -First instruction of OS is loaded into memory and address passed to PC (1 per -, max 4) [4] 2 (a) -during the fetch stage the contents of the address in the MAR is copied into the MDR … -The contents of MDR is the instruction to be used (this is copied to the CIR) -during the execute stage (store instruction) the contents of the ACC is copied into the MDR -during the execute stage (load/add instruction) the contents of the memory location is

copied into the MDR (1 per -, max 3) [3] (b) -Data bus carries contents of a memory location/contents of a register/a data value/an

address/an instruction -Address bus carries an address of a memory location/device -the address bus carries an address from the processor to main memory/a device -Control bus - Separate wires each used to carry a control signal - the bus carries control signals to the various components - by example e.g. read operation completed // interrupt -Data bus is bi-directional // data bus used to read/write data // Address bus is uni-directional

// the control bus is bi-directional (mark as 3 × 2 per bus, max 6) [6]

M.Mushtaq Hussain

Contact the Teacher: 03215275281

Page 46: UNIVERSITY OF CAMBRIDGE INTERNATIONAL EXAMINATIONS …iteach.pk/wp-content/uploads/2015/02/computer-science-papers-A1-… · COMPUTING 9691/31 Paper 3 May/June 2011 2 hours ... 10

Page 3 Mark Scheme: Teachers’ version Syllabus Paper

GCE A LEVEL – October/November 2011 9691 33

© University of Cambridge International Examinations 2011

3 (a) (i) +39 = 00100111 (ii) –47 = 10101111 (1 for both sign bits and 1 for each magnitude part) [3] (b) (i) –3 = 1111 1101 [2] (ii) –47 = 1101 0001 [2] (in each case 1 mark per nybble) (c) (i) = (1/8 + 1/32) * 2^4 = 5/32 *16 = 2½ OR: = 0.00101 * 2^4 Hence move point 4 places = 10.1 = 2 1/2 (1 mark for each underlined section, max 2. Note: Accept decimal values) [2] (ii) 010100 0010 (1 for mantissa, 1 for exponent) [2] (iii) M = ½ + ¼ + 1/32 OR = 25/32 E = 3 Number is 25/32 * 8 = 6 ¼ (1 per line, max 3) [3] 4 (a) -networked communication system...// content provided by a web server -probably provided on the Internet -Restricted access... -to specific members authorised by the health system -Access is password controlled -Content viewed using browser software (1 per -, max 3) [3] (b) Advantages -Limited number of users speeds up access... -Information being communicated is sensitive/confidential... -needs protection from being seen by unauthorised people -Information on system will be relevant/easily updated -Less information makes it easier to navigate -easier to control who can access the content Disadvantages -may involve additional set-up costs // need to set up a LAN -extra administration // setting up users (& passwords)/access rights (1 per -, max 5) [5]

M.Mushtaq Hussain

Contact the Teacher: 03215275281

Page 47: UNIVERSITY OF CAMBRIDGE INTERNATIONAL EXAMINATIONS …iteach.pk/wp-content/uploads/2015/02/computer-science-papers-A1-… · COMPUTING 9691/31 Paper 3 May/June 2011 2 hours ... 10

Page 4 Mark Scheme: Teachers’ version Syllabus Paper

GCE A LEVEL – October/November 2011 9691 33

© University of Cambridge International Examinations 2011

5 (a) -Cost of creating the real thing == different braking units would need to be built -Time that would be needed to create the real thing == the parameters of the simulated

braking system can be changed immediately -Time taken to run the tests == test time can be greatly reduced -a wide variety of conditions need to be tested == e.g. it may be necessary to drive for

100000 miles/at different speeds, this could be simulated -Ability to change conditions immediately == e.g. not necessary to transport car to different

parts of the world -extreme case scenarios can be tested == conditions may never occur during real life testing (3 x 2 per advantage) [6] (b) -Speed -in order to simulate stopping distances -Weight of car and load… -in order to simulate the effects of inertia with different loads -Materials used/size of structure used/methods of fixing to car -to try to reduce final cost while still remaining efficient -driving style -gentle braking/hard braking/cornering/reaction time -tyres -wear/type of tread/tyre material -road surface -roughness/material -weather conditions (temperature, wind, precipitation) -in order to replicate different climates… (1 per -, max 5) [5] 6 (a) -Reduces repetition/duplication of data items // keeps physical volume of data to a minimum

// minimises redundant data -Increases data integrity // reduces data inconsistency -Simpler data retrieval through queries // reports are easy to generate -Amending/searching/sorting data is easier -Amending the data structures is simpler to implement -Changes to the data structure will not affect existing applications programs // Program/data

independence (1 per -, max 3) [3] (b) (i) -GuestID or similar -unique [2] (ii) -e.g. Type of charge (bar/restaurant/laundry/…) -so that items can be accessed according to a different criteria other than by primary key

[2] (iii) -Attribute/field in one table which links to the primary key in another table [1] (iv) -GuestID… -to link each account to the relevant guest // to link ACCOUNT and GUEST tables [2]

M.Mushtaq Hussain

Contact the Teacher: 03215275281

Page 48: UNIVERSITY OF CAMBRIDGE INTERNATIONAL EXAMINATIONS …iteach.pk/wp-content/uploads/2015/02/computer-science-papers-A1-… · COMPUTING 9691/31 Paper 3 May/June 2011 2 hours ... 10

Page 5 Mark Scheme: Teachers’ version Syllabus Paper

GCE A LEVEL – October/November 2011 9691 33

© University of Cambridge International Examinations 2011

7 (a) -during lexical analysis … -identifiers and keywords are differentiated -checked against rules (e.g. length) for identifiers -Error messages produced if identifier does not match the expected rules -variable identifiers will be tokenised -variable identifiers entered into symbol table -Data type will be added to the entry in the symbol table -addresses in memory allocated to variables -during syntax/semantic analysis stage …. -assignment of illegal types of data to variables is reported (1 per -, max 7) [7] (b) (i) -object code is difficult to interfere with -object code runs faster than interpreted source code -compiler can optimise executable code -the code is not translated each time the program is run -Compiler does not need to be present when the program is run -Compiled code will be free from syntax errors (1 per -, max 2) [2] (ii) -Errors are more easily located… -reports errors when source code is present... -stopping at the point of the error -Parts (only) of program can be tested/testing can be started before all the program is

written -errors when found can be immediately corrected. (1 per -, max 2) [2] 8 Paging -memory is divided into equal-sized units called page frames -program/data file is divided into equal-size units called pages -one or more pages may be loaded into memory at any one time -Pages may be discontiguous -Pages swapped in and out as required -pages not in main memory are stored in virtual memory/backing store -page table/Index of pages/processes kept -absolute address is calculated by adding page address to relative address in instruction -paging is transparent to the programmer Segmentation -Memory is divided into variable length blocks -Programs can consist of many segments -Segments normally match natural divide in jobs/logical blocks -Index of segments stored which must… -store base address and length of segment -programmer will organise code modules into segments (1 per -, max 4 per dotty, max 6) [6]

M.Mushtaq Hussain

Contact the Teacher: 03215275281

Page 49: UNIVERSITY OF CAMBRIDGE INTERNATIONAL EXAMINATIONS …iteach.pk/wp-content/uploads/2015/02/computer-science-papers-A1-… · COMPUTING 9691/31 Paper 3 May/June 2011 2 hours ... 10

Page 6 Mark Scheme: Teachers’ version Syllabus Paper

GCE A LEVEL – October/November 2011 9691 33

© University of Cambridge International Examinations 2011

9 (a) (i) -describes what is to be accomplished -not how (no algorithm written) -the user states what is to be found/set a goal -Consists of a set of facts and rules -Rules are applied to the data until the goal is reached -Mention of backtracking/instantiation (max 1) (ii) -Program describes how to solve the problem in a sequence of steps/algorithm -lends itself to top-down design/modularisation -using procedures/functions (max 1) [2] (b) (i) -A class is the “blueprint” from which objects are defined // a class consists of the

properties and methods that define each object -Plant/Tree/Bulb is a class [2] (ii) -One class can use the properties and methods from a parent/base/super class -Tree/Bulb inherits the properties and methods of Plant [2] (iii) -An object can only read/write a property value using methods of the class // Class

contains both properties and the methods to use it -e.g. The CountryOfOrigin property can only be output using the getCountryOfOrigin

method in the class Tree [2] 10 (a) (i) Must begin with at least one <letter> (ii) X is not defined as a <letter> (iii) A maximum of 2 digits is allowed at the end [3] (b) Mark Points: -Only one entry and one exit point used -Order correct (letter, non-zero-digit, digit) -Loop around letter -alternative path to omit number -alternative path to omit 2nd digit (1 per -, max 4) [4]

non-zero-digit

identity-code

letter digit

M.Mushtaq Hussain

Contact the Teacher: 03215275281

Page 50: UNIVERSITY OF CAMBRIDGE INTERNATIONAL EXAMINATIONS …iteach.pk/wp-content/uploads/2015/02/computer-science-papers-A1-… · COMPUTING 9691/31 Paper 3 May/June 2011 2 hours ... 10

This document consists of 16 printed pages.

IB12 06_9691_32/FP © UCLES 2012 [Turn over

*6373916302*

UNIVERSITY OF CAMBRIDGE INTERNATIONAL EXAMINATIONS

General Certificate of Education Advanced Level

COMPUTING 9691/32

Paper 3 May/June 2012

2 hours

Candidates answer on the Question Paper.

No additional materials are required.

No calculators allowed.

READ THESE INSTRUCTIONS FIRST

Write your Centre number, candidate number and name on all the work you hand in.

Write in dark blue or black pen.

You may use a soft pencil for any diagrams, graphs or rough working.

Do not use staples, paper clips, highlighters, glue or correction fluid.

DO NOT WRITE IN ANY BARCODES.

Answer all questions.

No marks will be awarded for using brand names for software packages or hardware.

At the end of the examination, fasten all your work securely together.

The number of marks is given in brackets [ ] at the end of each question or part question.

M.Mushtaq Hussain

Contact the Teacher: 03215275281

Page 51: UNIVERSITY OF CAMBRIDGE INTERNATIONAL EXAMINATIONS …iteach.pk/wp-content/uploads/2015/02/computer-science-papers-A1-… · COMPUTING 9691/31 Paper 3 May/June 2011 2 hours ... 10

2

© UCLES 2012 9691/32/M/J/12

For

Examiner's

Use

1 A database is designed to store data about students at a college and the subjects that they study.

• All students are based in a tutor group

• A tutor supervises all the students in their tutor group

• Each subject has one subject teacher only

• Students study a number of subjects The following table StudentSubjects was a first attempt at the database design.

Table: StudentSubjects

StudentName TutorGroup Tutor Subject Level SubjectTeacher

Physics A SAN

Chemistry A MEB

Tom 6 SAN

Gen. Studies AS DIL

Geography AS ROG Joe 7 MEB

French AS HEN

Computing A VAR

Chemistry A MEB

Maths A COR

Samir 6 SAN

Gen. Studies A DIL

(a) (i) Explain why the table is not in First Normal Form (1NF).

[1]

(ii) Explain your answer by referring to the above data.

[1]

(b) The design is changed to the following: Student (StudentName, TutorGroup, Tutor)

StudentSubjectChoices (StudentName, Subject, Level, SubjectTeacher)

M.Mushtaq Hussain

Contact the Teacher: 03215275281

Page 52: UNIVERSITY OF CAMBRIDGE INTERNATIONAL EXAMINATIONS …iteach.pk/wp-content/uploads/2015/02/computer-science-papers-A1-… · COMPUTING 9691/31 Paper 3 May/June 2011 2 hours ... 10

3

© UCLES 2012 9691/32/M/J/12 [Turn over

For

Examiner's

Use

Using the data given in the original table, show how this data is now stored in the revised table designs.

Table: Student

StudentName TutorGroup Tutor

Table: StudentSubjectChoices

StudentName Subject Level SubjectTeacher

[3] (c) (i) Explain what is meant by a primary key.

[2]

(ii) A student is not allowed to choose the same subject at A Level and AS. What is the primary key of table StudentSubjectChoices?

[1]

M.Mushtaq Hussain

Contact the Teacher: 03215275281

Page 53: UNIVERSITY OF CAMBRIDGE INTERNATIONAL EXAMINATIONS …iteach.pk/wp-content/uploads/2015/02/computer-science-papers-A1-… · COMPUTING 9691/31 Paper 3 May/June 2011 2 hours ... 10

4

© UCLES 2012 9691/32/M/J/12

For

Examiner's

Use

(iii) There is a relationship between tables Student and StudentSubjectChoices.

Explain how the relationship is established using a primary key and foreign key.

[2]

(d) The design of table StudentSubjectChoices is:

StudentSubjectChoices (StudentName, Subject, Level, SubjectTeacher)

Explain why this table is not in Second Normal Form (2NF).

[2]

(e) The design of table Student is:

Student (StudentName, TutorGroup, Tutor)

Explain why this table is not in Third Normal Form (3NF).

[2]

M.Mushtaq Hussain

Contact the Teacher: 03215275281

Page 54: UNIVERSITY OF CAMBRIDGE INTERNATIONAL EXAMINATIONS …iteach.pk/wp-content/uploads/2015/02/computer-science-papers-A1-… · COMPUTING 9691/31 Paper 3 May/June 2011 2 hours ... 10

5

© UCLES 2012 9691/32/M/J/12 [Turn over

For

Examiner's

Use

2 A binary pattern can be used to represent different data used in a computer system. (a) Consider the binary pattern: 0101 0011 The pattern represents an integer. What number is this in denary?

[1]

(b) Consider the binary pattern: 0001 0101 0011 The pattern represents a Binary Coded Decimal (BCD) number. What number is this in denary?

[1]

(c) Consider the binary pattern: 1001 0010 This represents a two’s complement integer. What number is this in denary?

[1]

M.Mushtaq Hussain

Contact the Teacher: 03215275281

Page 55: UNIVERSITY OF CAMBRIDGE INTERNATIONAL EXAMINATIONS …iteach.pk/wp-content/uploads/2015/02/computer-science-papers-A1-… · COMPUTING 9691/31 Paper 3 May/June 2011 2 hours ... 10

6

© UCLES 2012 9691/32/M/J/12

For

Examiner's

Use

(d) Floating point is to be used to represent real numbers with:

• 8 bits for the mantissa, followed by

• 4 bits for the exponent

• two’s complement used for both the mantissa and the exponent (i) Consider the binary pattern:

0 1 1 0 1 0 0 0 0 1 0 0

What number is this in denary? Show your working.

[3]

(ii) The representation shown in part (d)(i) is normalised. Explain why floating point numbers are normalised.

[1]

(iii) Show the binary pattern for the smallest positive number which can be stored

using a normalised 12-bit floating point representation. Mantissa:

Exponent:

Work out its denary value.

Denary: [3]

M.Mushtaq Hussain

Contact the Teacher: 03215275281

Page 56: UNIVERSITY OF CAMBRIDGE INTERNATIONAL EXAMINATIONS …iteach.pk/wp-content/uploads/2015/02/computer-science-papers-A1-… · COMPUTING 9691/31 Paper 3 May/June 2011 2 hours ... 10

7

© UCLES 2012 9691/32/M/J/12 [Turn over

For

Examiner's

Use

(e) The developer of a new programming language decides that all real numbers will be stored using 20-bit normalised floating point representation. She cannot decide how many bits to use for the mantissa and how many for the exponent.

Explain the trade-off between using either a large number of bits for the mantissa, or a

large number of bits for the exponent.

[2]

M.Mushtaq Hussain

Contact the Teacher: 03215275281

Page 57: UNIVERSITY OF CAMBRIDGE INTERNATIONAL EXAMINATIONS …iteach.pk/wp-content/uploads/2015/02/computer-science-papers-A1-… · COMPUTING 9691/31 Paper 3 May/June 2011 2 hours ... 10

8

© UCLES 2012 9691/32/M/J/12

For

Examiner's

Use

3 (a) Customer names are stored in the array Customer.

An algorithm is to be designed to perform a serial search of the array for a requested customer name.

The algorithm will use the variables shown in the table. Study the table and the algorithm and fill in the gaps.

Identifier Data Type Description

Customer ARRAY[2000] OF STRING The customer names

Index INTEGER Index position in the customer array

IsFound

SearchName STRING The requested customer name

//Serial search algorithm

INPUT

IsFound ← FALSE

Index ← 1

REPEAT

IF Customer [ ] = SearchName

THEN

IsFound ← TRUE

OUTPUT “FOUND – at position ” Index “ in the array”

ELSE

Index ←

ENDIF

UNTIL (IsFound = TRUE) OR

IF

THEN

OUTPUT “Customer name was NOT FOUND”

ENDIF [7] (b) Comment on the efficiency of the serial search algorithm in part (a) for retrieving a data

item from an array with 2000 items.

[2]

M.Mushtaq Hussain

Contact the Teacher: 03215275281

Page 58: UNIVERSITY OF CAMBRIDGE INTERNATIONAL EXAMINATIONS …iteach.pk/wp-content/uploads/2015/02/computer-science-papers-A1-… · COMPUTING 9691/31 Paper 3 May/June 2011 2 hours ... 10

9

© UCLES 2012 9691/32/M/J/12 [Turn over

For

Examiner's

Use

(c) A binary search may be an alternative algorithm to a serial search. (i) Describe how this algorithm works. (Do not attempt to write the pseudocode.)

[4]

(ii) A binary search is made to locate Cherry.

1 Apple

2 Banana

3 Cherry

4 Kiwi

5 Lemon

6 Mango

7 Plum

List, in order, the comparisons which are made.

[3]

M.Mushtaq Hussain

Contact the Teacher: 03215275281

Page 59: UNIVERSITY OF CAMBRIDGE INTERNATIONAL EXAMINATIONS …iteach.pk/wp-content/uploads/2015/02/computer-science-papers-A1-… · COMPUTING 9691/31 Paper 3 May/June 2011 2 hours ... 10

12

© UCLES 2012 9691/32/M/J/12

For

Examiner's

Use

5 The table shows the assembly language instructions for a processor which has one general purpose register – the Accumulator.

Instruction

Op Code Operand Explanation

LDD <address> Load using direct addressing

STO <address> Store the contents of the Accumulator at the given address

LDI <address> Load using indirect addressing

LDX <address> Load using indexed addressing

INC Add 1 to the contents of the Accumulator

END End the program and return to the operating system

(a) Write on the diagram to explain the instruction shown. Show the contents of the Accumulator after the execution of the instruction.

LDD 105 Main memory

100 0100 0000

101 0110 1000

Accumulator 102 1111 1110

103 1111 1010

104 0101 1101

105 0001 0001

106 1010 1000

107 1100 0001

200 1001 1111

[2] (b) Write on the diagram to explain the instruction shown. Show the contents of the registers after the execution of the instruction.

LDX 101 Main memory

100 0100 0000

101 0110 1000

Accumulator 102 1111 1110

103 1111 1010

104 0101 1101

105 0001 0001

Index Register 106 1010 1000

0000 0011 107 1100 0001

200 1001 1111

[4]

M.Mushtaq Hussain

Contact the Teacher: 03215275281

Page 60: UNIVERSITY OF CAMBRIDGE INTERNATIONAL EXAMINATIONS …iteach.pk/wp-content/uploads/2015/02/computer-science-papers-A1-… · COMPUTING 9691/31 Paper 3 May/June 2011 2 hours ... 10

13

© UCLES 2012 9691/32/M/J/12 [Turn over

For

Examiner's

Use

(c) Trace this assembly language program using the trace table below.

500 LDD 507

501 INC

502 STO 509

503 LDD 508

504 INC

505 STO 510

506 END

507 22

508 170

509 0

510 0

Memory Address

Accumulator 507 508 509 510

22 170 0 0

[5] (d) Explain the relationship between assembly language instructions and machine code

instructions.

[1]

M.Mushtaq Hussain

Contact the Teacher: 03215275281

Page 61: UNIVERSITY OF CAMBRIDGE INTERNATIONAL EXAMINATIONS …iteach.pk/wp-content/uploads/2015/02/computer-science-papers-A1-… · COMPUTING 9691/31 Paper 3 May/June 2011 2 hours ... 10

14

© UCLES 2012 9691/32/M/J/12

For

Examiner's

Use

6 In a multiprogramming environment the operating system includes a scheduler. (a) Explain the purpose of the scheduler.

[2]

(b) A process will at any time be in one of three states. (i) Name and describe each possible state.

1

2

3

[6]

(ii) How will the operating system keep details about the state of all processes?

[1]

M.Mushtaq Hussain

Contact the Teacher: 03215275281

Page 62: UNIVERSITY OF CAMBRIDGE INTERNATIONAL EXAMINATIONS …iteach.pk/wp-content/uploads/2015/02/computer-science-papers-A1-… · COMPUTING 9691/31 Paper 3 May/June 2011 2 hours ... 10

Page 2 Mark Scheme: Teachers’ version Syllabus Paper

GCE A LEVEL – May/June 2012 9691 32

© University of Cambridge International Examinations 2012

1 (a) (i) The table/each student has a repeated group of attributes // each student has a number of subjects [1]

(ii) StudentName, TutorGroup and Tutor would need to be repeated for each record [1] (b) Table: Student Table: StudentSubjectChoices

StudentName TutorGroup Tutor Student Name

Subject Level Subject Teacher

Tom 6 SAN Tom Physics A SAN

Joe 7 MEB Tom Chemistry A MEB

Samir 6 SAN Tom Gen Studies AS DIL

Joe Geography AS ROG

Joe French AS HEN

Samir Computing A VAR

Samir Chemistry A MEB

Samir Maths A COR

Samir Gen. Studies A DIL

Mark as follows .... Complete Student table [1] Repetition of StudentName in StudentSubjectchoices table [1] Complete columns 2, 3, and 4 [1]

(c) (i) primary key...

- an attribute/combination of attributes - chosen to ensure that the records in a table are unique // used to identify a record/tuple [2]

(ii) StudentName + Subject Correct Answer Only [1] (iii) - there is a one-to-many relationship // Student is the ‘one side’ table –

StudentSubjectChoices is the ‘many side‘ table. - The primary key (attribute StudentName) in Student - Links to StudentName in the StudentSubjectChoices table - (StudentName in the) StudentSubjectChoices table is the foreign key // StudentName is the foreign key that links the two tables [MAX 2]

(d) - There are non-key attributes ...

- SubjectTeacher ... - dependent only on part of the primary key (i.e. Subject) // partial dependency [MAX 2]

(e) - There are dependent non-key attributes // there are non-key dependencies

- TutorGroup is dependant on Tutor // Tutor is dependent on TutorGroup [2] [Total: 14] 2 (a) 83 [1] (b) 153 [1]

M.Mushtaq Hussain

Contact the Teacher: 03215275281

Page 63: UNIVERSITY OF CAMBRIDGE INTERNATIONAL EXAMINATIONS …iteach.pk/wp-content/uploads/2015/02/computer-science-papers-A1-… · COMPUTING 9691/31 Paper 3 May/June 2011 2 hours ... 10

Page 3 Mark Scheme: Teachers’ version Syllabus Paper

GCE A LEVEL – May/June 2012 9691 32

© University of Cambridge International Examinations 2012

(c) –110 [1] (d) (i) +13

mark as follows: Exponent: +4 // move the pattern four places Mantissa: +13/16 // 0.1101 Answer: 13/16 × 24 // or equivalent [3]

(ii) There will be a unique representation for a number

The format will ensure the number is represented with the greatest possible/more accuracy/precision Multiplication is performed more accurately/precisely [MAX 1]

(iii) Mantissa: 0100 0000

Exponent: 1000 Therefore number is ½ * 2–8 // +1/512 // +2–9 // 0.00195 [3]

(e) choices made will effect range and accuracy

More bits used for the mantissa will result in better accuracy More bits use for the exponent will result in larger range of numbers [Max 2]

[Total: 12] 3 (a) Boolean [1]

Flags whether or not the requested customer name is found [1] SearchName [1]

Index [1]

Index + 1 [1]

Index = 2001 // Index >= 2001 // Index > 2000 [1]

IsFound = FALSE // NOT IsFound // Index = 2001 // Index > 2000 [1]

(b) - values are considered in sequence

- when an item is not found all items are considered - Few comparisons are needed if the value is near the start of the list // Many comparisons are needed/it’s time consuming if the value is near the end of the list - The average number of comparisons needed will be N/2 (or 1000 for this data set) [MAX 2]

(c) (i) The values must be in order Calculate the middle value and compare with the requested value If Requested value is less/greater discard the top/bottom list Repeat with a new list // compare with a new middle value Continue until value is found or list is empty [MAX 4]

(ii) Compare with ...

Kiwi Banana Cherry [3]

[Total: 16]

M.Mushtaq Hussain

Contact the Teacher: 03215275281

Page 64: UNIVERSITY OF CAMBRIDGE INTERNATIONAL EXAMINATIONS …iteach.pk/wp-content/uploads/2015/02/computer-science-papers-A1-… · COMPUTING 9691/31 Paper 3 May/June 2011 2 hours ... 10

Page 4 Mark Scheme: Teachers’ version Syllabus Paper

GCE A LEVEL – May/June 2012 9691 32

© University of Cambridge International Examinations 2012

4 (a) 21 [1]

(b) (i) a 5 – b c + / [1]

(ii) 2 3 * 6 2 / + [2]

(c) Expressions can be evaluated without the use of brackets Operators are in execution order / No need to apply a precedence of operators [1]

(d) (i) Last item added to the stack will be the first item to leave [1]

(ii) Static structure The size of the array will be fixed / size will be defined before the array is used [2]

(iii)

5

4

3

2 7 6 9

1 3 3 21 or 21 27 27 3

1 1 1 1 [4]

[Total: 12]

5 (a) LDD 105 Main memory

100 0100 0000 101 0110 1011 Accumulator 102 1111 1110

0001 0001 103 1111 1010

104 0101 1101 105 0001 0001 106 1010 1000 107 1100 0001

200 1001 1111

Mark as follows: - Sensible annotation which makes clear 105 is the address used - Final value in Acc [2]

M.Mushtaq Hussain

Contact the Teacher: 03215275281

Page 65: UNIVERSITY OF CAMBRIDGE INTERNATIONAL EXAMINATIONS …iteach.pk/wp-content/uploads/2015/02/computer-science-papers-A1-… · COMPUTING 9691/31 Paper 3 May/June 2011 2 hours ... 10

Page 5 Mark Scheme: Teachers’ version Syllabus Paper

GCE A LEVEL – May/June 2012 9691 32

© University of Cambridge International Examinations 2012

(b) LDX 101 Main memory

100 0100 0000 101 0110 1011 Accumulator 102 1111 1110

0101 1101 103 1111 1010

104 0101 1101

105 0001 0001 Index Register 106 1010 1000

00000011 107 1100 0001

200 1001 1111

Mark as follows: - IR contents converted to 3 - Computed address of 101 + 3 = 104 // explanation: add contents of IR to address part of instruction - Then, ‘direct addressing’ to 104 - Final value in Acc [MAX 4]

(c)

Memory Address

Accumulator 507 508 509 510

22 170 0 0

22

23

23

170

171

171

Mark as follows ... - 22 to Accumulator - Incremented to 23 - 23 copied to address 509 - 170 copied to Accumulator and incremented to 171 - 171 in address 510 [5]

(d) Every assembly language instruction is translated into exactly one machine code instruction /

there is a 1-to-1 relationship between them [1] [Total: 11]

M.Mushtaq Hussain

Contact the Teacher: 03215275281

Page 66: UNIVERSITY OF CAMBRIDGE INTERNATIONAL EXAMINATIONS …iteach.pk/wp-content/uploads/2015/02/computer-science-papers-A1-… · COMPUTING 9691/31 Paper 3 May/June 2011 2 hours ... 10

Page 6 Mark Scheme: Teachers’ version Syllabus Paper

GCE A LEVEL – May/June 2012 9691 32

© University of Cambridge International Examinations 2012

6 (a) Decide which process ... Gets next use of the processor (low level scheduler) // is next loaded into memory (high level scheduler) maximise system resources [2]

(b) (i) Running

The process currently has the use of the processor Runnable/Ready The process would like to use the processor but the processor is currently in use by another process Suspended/Blocked The process is not capable of using the processor / the process is currently occupied doing I/O [6]

(ii) Maintain a separate ‘data structure’ for the processes in each state

one field of the Process Control Block will store the current state [1] (c) (i) Processor bound ...

The process does very little I/O // the process requires the processor most of the time 3D-graphics calculation // any plausible application I/O bound ... The process does lots of I/O // the process requires little processor time // any plausible application [4]

(ii) Priority to I/O bound processes

Otherwise they will not get a look in // processor bound jobs would monopolise the processor [2]

[Total: 15] 7 (a) a model/program of the real-world system is produced

to predict the likely behaviour of a real-world system [2] (b) Computer system suitable as ...

A computer program/system can be written/created which model the problem/application The problem can control the values of all the variables/parameters The computer can produce results very quickly // e.g. models what actually takes several days into 5 minutes processing The simulation removes any element of hazard/danger Some real-world problems are impossible to create It will be cost-effective to model the problem first [MAX 2]

M.Mushtaq Hussain

Contact the Teacher: 03215275281

Page 67: UNIVERSITY OF CAMBRIDGE INTERNATIONAL EXAMINATIONS …iteach.pk/wp-content/uploads/2015/02/computer-science-papers-A1-… · COMPUTING 9691/31 Paper 3 May/June 2011 2 hours ... 10

Page 7 Mark Scheme: Teachers’ version Syllabus Paper

GCE A LEVEL – May/June 2012 9691 32

© University of Cambridge International Examinations 2012

(c) Time taken to serve a customer Number of items in the customer basket Acceptable wait time in the queue Number of checkouts Time of day/day of the week Number of customers arriving Speed of the checkout operators Anything plausible ... [MAX 3]

(d) - Increase the average time taken to serve a customer

- … will increase the average queue length Or anything plausible ... [2]

[Total: 9]

M.Mushtaq Hussain

Contact the Teacher: 03215275281

Page 68: UNIVERSITY OF CAMBRIDGE INTERNATIONAL EXAMINATIONS …iteach.pk/wp-content/uploads/2015/02/computer-science-papers-A1-… · COMPUTING 9691/31 Paper 3 May/June 2011 2 hours ... 10

This document consists of 20 printed pages.

IB13 06_9691_32/FP © UCLES 2013 [Turn over

*3270941184*

UNIVERSITY OF CAMBRIDGE INTERNATIONAL EXAMINATIONS

General Certificate of Education Advanced Level

COMPUTING 9691/32

Paper 3 May/June 2013

2 hours

Candidates answer on the Question Paper.

No additional materials are required.

No calculators allowed.

READ THESE INSTRUCTIONS FIRST

Write your Centre number, candidate number and name on all the work you hand in.

Write in dark blue or black pen.

You may use a soft pencil for any diagrams, graphs or rough working.

Do not use staples, paper clips, highlighters, glue or correction fluid.

DO NOT WRITE IN ANY BARCODES.

Answer all questions.

No marks will be awarded for using brand names for software packages or hardware.

At the end of the examination, fasten all your work securely together.

The number of marks is given in brackets [ ] at the end of each question or part question.

M.Mushtaq Hussain

Contact the Teacher: 03215275281

Page 69: UNIVERSITY OF CAMBRIDGE INTERNATIONAL EXAMINATIONS …iteach.pk/wp-content/uploads/2015/02/computer-science-papers-A1-… · COMPUTING 9691/31 Paper 3 May/June 2011 2 hours ... 10

2

© UCLES 2013 9691/32/M/J/13

For

Examiner's

Use

1 A hospital has a number of doctors and a number of wards. A ward has a number of patients. A patient may be treated by more than one doctor and a doctor treats many patients.

Data is to be recorded in a relational database and the tables include DOCTOR and

PATIENT. (a) (i) What is the relationship between DOCTOR and PATIENT?

[1]

(ii) Show this relationship with an entity-relationship (E-R) diagram. [1] (iii) Draw an E-R diagram showing a database design which can be produced so that

the doctor and patient data are fully normalised. Explain how the relationships are implemented.

[4]

(b) (i) What is the relationship between WARD and PATIENT?

[1]

(ii) Show this relationship with an E-R diagram. [1]

M.Mushtaq Hussain

Contact the Teacher: 03215275281

Page 70: UNIVERSITY OF CAMBRIDGE INTERNATIONAL EXAMINATIONS …iteach.pk/wp-content/uploads/2015/02/computer-science-papers-A1-… · COMPUTING 9691/31 Paper 3 May/June 2011 2 hours ... 10

3

© UCLES 2013 9691/32/M/J/13 [Turn over

For

Examiner's

Use

(c) Two of the incomplete table designs are: PATIENT(PatientID, PatientName, Address, NextOfKin)

WARD(WardName, NurseInCharge, NumberOfBeds,

NumberOfOccupiedBeds, Specialism)

Explain how the relationship between PATIENT and WARD is implemented.

[2]

(d) The following Data Manipulation Language query is run. SELECT WardName

FROM WARD

WHERE NumberOfOccupiedBeds < NumberOfBeds

What useful information is produced for the Hospital Administrator?

[2]

M.Mushtaq Hussain

Contact the Teacher: 03215275281

Page 71: UNIVERSITY OF CAMBRIDGE INTERNATIONAL EXAMINATIONS …iteach.pk/wp-content/uploads/2015/02/computer-science-papers-A1-… · COMPUTING 9691/31 Paper 3 May/June 2011 2 hours ... 10

6

© UCLES 2013 9691/32/M/J/13

For

Examiner's

Use

3 The table shows the assembly language instructions for a processor which has one general purpose register – the Accumulator (ACC), and an index register (IX).

Instruction

Op Code Operand Explanation

LDD <address> Direct addressing. Load the contents of the given address to ACC

STO <address> Store the contents of ACC at the given address

LDI <address> Indirect addressing. At the given address is the address to be used. Load the contents of this second address to ACC

LDX <address> Indexed addressing. Form the address as <address> +

the contents of IX. Copy the contents of this address to ACC

INC <register> Add 1 to the contents of the register (ACC or IX)

ADD <address> Add the contents of the given address to the contents of ACC

OUT Output the contents of ACC (as a denary number) to the monitor

IN Input a denary number from the keyboard and store in ACC

END End the program and return to the operating system

M.Mushtaq Hussain

Contact the Teacher: 03215275281

Page 72: UNIVERSITY OF CAMBRIDGE INTERNATIONAL EXAMINATIONS …iteach.pk/wp-content/uploads/2015/02/computer-science-papers-A1-… · COMPUTING 9691/31 Paper 3 May/June 2011 2 hours ... 10

7

© UCLES 2013 9691/32/M/J/13 [Turn over

For

Examiner's

Use

The diagram shows a program loaded in main memory starting at location 100. Two of the op-codes have been partially blanked out. Locations 200 onwards contain data which is used by the program. (a) The instruction at address 100 is fetched. Shown are the contents of the registers after

execution.

100 LD■ 202

101 INC ACC

ACC 102 INC ACC

88 103 LD■ 203

104 INC ACC

105 LDI 203

IX 106 INC ACC

2 107 END

200 38

201 205

202 88

203 200

204 48

205 126

Which mode of addressing was used by this load instruction at address 100?

[1]

(b) The instruction at address 103 is fetched. Shown are the contents of the registers after

execution.

ACC

126

IX

2

Which mode of addressing was used by this load instruction at address 103?

[1]

M.Mushtaq Hussain

Contact the Teacher: 03215275281

Page 73: UNIVERSITY OF CAMBRIDGE INTERNATIONAL EXAMINATIONS …iteach.pk/wp-content/uploads/2015/02/computer-science-papers-A1-… · COMPUTING 9691/31 Paper 3 May/June 2011 2 hours ... 10

8

© UCLES 2013 9691/32/M/J/13

For

Examiner's

Use

(c) The instruction at address 105 is fetched and executed. Draw on the diagram to explain how this instruction is executed and show the contents

of ACC after execution.

100 LD■ 202

101 INC ACC

ACC 102 INC ACC

103 LD■ 203

104 INC ACC

105 LDI 203

IX 106 INC ACC

107 END

200 38

201 205

202 88

203 200

204 48

205 126

[2]

M.Mushtaq Hussain

Contact the Teacher: 03215275281

Page 74: UNIVERSITY OF CAMBRIDGE INTERNATIONAL EXAMINATIONS …iteach.pk/wp-content/uploads/2015/02/computer-science-papers-A1-… · COMPUTING 9691/31 Paper 3 May/June 2011 2 hours ... 10

9

© UCLES 2013 9691/32/M/J/13 [Turn over

For

Examiner's

Use

(d) Trace this assembly language program using the trace table below. The user inputs 19 followed by 37.

100 IN

101 INC ACC

102 STO 109

103 IN

104 INC ACC

105 ADD 109

106 STO 110

107 OUT

108 END

Memory location

ACC 109 110 Output

[5] (e) In (d) the program was shown in assembly language. In practice this must be machine

code in order to execute the program. Explain how the assembler software translates a program from assembly language into

machine code.

[3]

M.Mushtaq Hussain

Contact the Teacher: 03215275281

Page 75: UNIVERSITY OF CAMBRIDGE INTERNATIONAL EXAMINATIONS …iteach.pk/wp-content/uploads/2015/02/computer-science-papers-A1-… · COMPUTING 9691/31 Paper 3 May/June 2011 2 hours ... 10

15

© UCLES 2013 9691/32/M/J/13 [Turn over

For

Examiner's

Use

(b) The final stage of compilation is code optimisation. (i) Explain what is meant by code optimisation.

[2]

(ii) Give one practical example of code which would benefit from optimising.

[1]

6 A multiprogramming, multi-user operating system organises the available memory into two

fixed sized partitions.

• Partition 1 – size 30 Gb – is used only for batch processing • Partition 2 – size 40 Gb – is used only for interactive processing at a terminal

A program, once loaded, occupies the same memory locations until its execution is

complete. (a) Explain what is meant by: (i) Batch processing

[3]

(ii) Interactive processing

[1]

M.Mushtaq Hussain

Contact the Teacher: 03215275281

Page 76: UNIVERSITY OF CAMBRIDGE INTERNATIONAL EXAMINATIONS …iteach.pk/wp-content/uploads/2015/02/computer-science-papers-A1-… · COMPUTING 9691/31 Paper 3 May/June 2011 2 hours ... 10

16

© UCLES 2013 9691/32/M/J/13

For

Examiner's

Use

(b) The diagram shows the current contents of main memory with a list of programs waiting to be scheduled.

OTHER

PROG8

PROG7

PROG6

PROG9

10 Gb

20 Gb

30 Gb

40 Gb

50 Gb

60 Gb

70 Gb

80 Gb

Partition 1

Partition 2

Currently waiting to be loaded are:

• PROG16 (batch processing requiring 25 Gb) • PROG17 (interactive processing requiring 8 Gb) • PROG44 (interactive processing requiring 9 Gb) • PROG45 (interactive processing requiring 8 Gb) • PROG23 (batch processing requiring 15 Gb)

(i) If PROG6 completes execution, which programs (if any) can be loaded next?

[1]

(ii) If PROG8 completes execution, which programs (if any) can be loaded next?

[1]

(iii) Describe two variations on this use of partitions which may make more efficient

use of memory.

1

2

[2]

M.Mushtaq Hussain

Contact the Teacher: 03215275281

Page 77: UNIVERSITY OF CAMBRIDGE INTERNATIONAL EXAMINATIONS …iteach.pk/wp-content/uploads/2015/02/computer-science-papers-A1-… · COMPUTING 9691/31 Paper 3 May/June 2011 2 hours ... 10

17

© UCLES 2013 9691/32/M/J/13 [Turn over

For

Examiner's

Use

(c) 10 Gb of the main memory is labelled OTHER and will not be used for the execution of application programs.

Name two possible items of software this memory will be used for.

1

2 [2]

(d) Any program loaded is always in one of three possible states. One is the ‘running’ state

meaning the job currently has use of the processor. Name and describe the two other states.

1

2

[4]

M.Mushtaq Hussain

Contact the Teacher: 03215275281

Page 78: UNIVERSITY OF CAMBRIDGE INTERNATIONAL EXAMINATIONS …iteach.pk/wp-content/uploads/2015/02/computer-science-papers-A1-… · COMPUTING 9691/31 Paper 3 May/June 2011 2 hours ... 10

18

© UCLES 2013 9691/32/M/J/13

For

Examiner's

Use

7 (a) A user-defined function CountCharacter is defined, using pseudocode, as follows:

FUNCTION CountCharacter(ThisArray : CHAR, UBound : INTEGER,

ThisCharacter : CHAR) RETURNS INTEGER

The function checks each element in the array ThisArray with upper bound UBound. The function returns the number of times ThisCharacter appears in ThisArray.

If the function is incorrectly formed it will give a ‘COMPILE ERROR’.

The function is used with the three arrays shown below: Subscript/Index

Identifier 1 2 3 4 5 6 7 8 9 10

City1 L O N D O N

City2 N E W <Space> Y O R K

City3 S T O C K H O L M

What is returned by the following function calls? (i) CountCharacter(City1, 6, ‘O’)

[1]

(ii) CountCharacter(City2, ‘R’, 8)

[1]

(iii) CountCharacter(City3, 9, ‘t’)

[1]

(iv) CountCharacter(City2, ’W’)

[1]

M.Mushtaq Hussain

Contact the Teacher: 03215275281

Page 79: UNIVERSITY OF CAMBRIDGE INTERNATIONAL EXAMINATIONS …iteach.pk/wp-content/uploads/2015/02/computer-science-papers-A1-… · COMPUTING 9691/31 Paper 3 May/June 2011 2 hours ... 10

19

© UCLES 2013 9691/32/M/J/13 [Turn over

For

Examiner's

Use

(b) A programmer writes pseudocode to report whether or not a string value ThisValue is found in the string array ThisArray. This is designed as a function StringFound

which returns a Boolean value. (i) Show the function header for function StringFound.

[3]

(ii) Use the following numbers to label your function header above.

1. Function parameter(s) 2. Where the return data type is stated [2]

(iii) Write a single statement which calls the function StringFound to search for the

value “LISBON” in the array CapitalCities (upper bound 300). The function returns the value to variable CityWasFound in the calling statement.

[2]

M.Mushtaq Hussain

Contact the Teacher: 03215275281

Page 80: UNIVERSITY OF CAMBRIDGE INTERNATIONAL EXAMINATIONS …iteach.pk/wp-content/uploads/2015/02/computer-science-papers-A1-… · COMPUTING 9691/31 Paper 3 May/June 2011 2 hours ... 10

Page 2 Mark Scheme Syllabus Paper

GCE A LEVEL – May/June 2013 9691 32

© Cambridge International Examinations 2013

1 (a) (i) Many-to-many [1] (ii) E-R diagram

[1] (iii)

Link table drawn [1]

2 × one-to-many relationships [1]

primary key in DOCTOR links to foreign key in link table [1] primary key in PATIENT links to foreign key in link table [1] No mention of foreign key scores max 1 for final two points …

(b) (i) One to many [1]

(ii) E-R diagram

[1]

(c) The primary key of table WARD - WardName [1] Matches to WardName in the PATIENT table [1]

(d) Displays a ‘list’ of the wards (names) [1] R. Number of wards Which has unoccupied beds available [1] R. the condition explained using the attribute identifiers

[Total: 12]

M.Mushtaq Hussain

Contact the Teacher: 03215275281

Page 81: UNIVERSITY OF CAMBRIDGE INTERNATIONAL EXAMINATIONS …iteach.pk/wp-content/uploads/2015/02/computer-science-papers-A1-… · COMPUTING 9691/31 Paper 3 May/June 2011 2 hours ... 10

Page 4 Mark Scheme Syllabus Paper

GCE A LEVEL – May/June 2013 9691 32

© Cambridge International Examinations 2013

3 (a) Direct addressing / LDD [1]

(b) Indexed addressing / LDX [1]

(c) Annotation to show 203 used as a forwarding address [1]

Accumulator contains 38 [1]

(d)

Memory location

ACC 109 110 Output

19 (must be the first column entry)

0

20 20

37

38

58 58 /ft 58 /ft

1 mark for each of the emboldened numbers in the correct column and sequence [MAX 5]

(e) Labels added to a (symbol) table // creates a list of addresses [1] Labels are later looked up to determine the actual address / Assembler must allocate addresses to labels [1] Mnemonic looked up to give binary code/machine code [1] Macro instructions are expanded into a group of instructions [1] The software makes two passes through the source program [1]

[MAX 3]

[Total: 12]

M.Mushtaq Hussain

Contact the Teacher: 03215275281

Page 82: UNIVERSITY OF CAMBRIDGE INTERNATIONAL EXAMINATIONS …iteach.pk/wp-content/uploads/2015/02/computer-science-papers-A1-… · COMPUTING 9691/31 Paper 3 May/June 2011 2 hours ... 10

Page 7 Mark Scheme Syllabus Paper

GCE A LEVEL – May/June 2013 9691 32

© Cambridge International Examinations 2013

5 (a) (i) The keyword table contains: all the language keywords/reserved words + with a matching token [1]

The symbol table stores:

each identifier/variable found (and its data type) [1] the values of all constants [1] the upper and lower bounds of arrays [1] [Mark as: 1 + 1]

[MAX 2]

(ii) Keywords are looked up in the keyword table [1]

Keywords are converted to tokens [1] Identifiers/Variables are looked up in the symbol table [1] Identifiers/variables are converted to actual addresses [1]

[MAX 2]

(iii) The white space // redundant characters are removed [1] Illegal identifier names are identified [1]

[MAX 1] (b) (i) Optimising

Code will execute/run/process faster [1] Code requires less memory Reduce the amount of code [1]

R. ‘more efficient’ // removes redundant code

(ii) Any example where the code could be changed [1]

E.g. input of a list of number to compute the total (There would be no need to store the numbers first)

[Total: 8]

M.Mushtaq Hussain

Contact the Teacher: 03215275281

Page 83: UNIVERSITY OF CAMBRIDGE INTERNATIONAL EXAMINATIONS …iteach.pk/wp-content/uploads/2015/02/computer-science-papers-A1-… · COMPUTING 9691/31 Paper 3 May/June 2011 2 hours ... 10

Page 8 Mark Scheme Syllabus Paper

GCE A LEVEL – May/June 2013 9691 32

© Cambridge International Examinations 2013

6 (a) (i) Batch processing All input/processing/output is performed as a batch [1] There may be a time delay before processing [1] All the (data) is processed together/at the same time [1] There is no user involvement [1] Processing will not start until all the data is available/collected [1]

[MAX 3]

(ii) Interactive processing The user is constantly interacting directly with the processor [1]

(b) (i) PROG23 [1]

(ii) Any two from PROG17, PROG44 and 45 [1]

(iii) Jobs do not have to occupy a continuous block of memory [1]

Move all jobs still loaded in the partition so that when a job completes there is only ever one ‘hole’ remaining [1] Make the partitions of variable size [1] Allow only part of a program to be initially loaded // paging //segmentation [1] [MAX 2]

(c) Operating system // specific modules e.g. interrupt handler/scheduler, etc [1]

device drivers [1] examples of system software or utilities [1] R. “System software” and “Utilities” [MAX 2]

(d) Runnable // Ready [1]

the program is capable of being run and is awaiting its turn for the use of the processor [1] R. explanation of (only) ‘ready to use the processor’ Suspended // Blocked [1] the program is unable use the processor/ or by example, the job is currently using an I/O device [1] Note: the explanation marks are not dependant on the correct name

[Total: 14]

M.Mushtaq Hussain

Contact the Teacher: 03215275281

Page 84: UNIVERSITY OF CAMBRIDGE INTERNATIONAL EXAMINATIONS …iteach.pk/wp-content/uploads/2015/02/computer-science-papers-A1-… · COMPUTING 9691/31 Paper 3 May/June 2011 2 hours ... 10

Page 9 Mark Scheme Syllabus Paper

GCE A LEVEL – May/June 2013 9691 32

© Cambridge International Examinations 2013

7 (a) (i) 2 [1]

(ii) COMPILE ERROR // reporting an error [1]

(iii) 0 [1]

(iv) COMPILE ERROR // reporting an error [1]

(b) (i) FUNCTION StringFound(ThisArray : STRING , UBound : INTEGER,

ThisValue : STRING) RETURNS BOOLEAN

Mark as follows: FUNCTION StringFound [1]

‘Array variable’ : STRING data type [1]

ThisValue : STRING // ‘UBound’ : INTEGER [1]

(ii) Numbered 1 – Parameter identifiers labelled [1]

Numbered 2 – (RETURNS) BOOLEAN [1]

(iii) CityWasFound = StringFound(CapitalCities, 300, “LISBON”)

Mark as follows: CityWasFound = StringFound( ... [1]

“LISBON” is the correct position (f/t from ‘their’ function header) [1]

[Total: 11]

M.Mushtaq Hussain

Contact the Teacher: 03215275281

Page 85: UNIVERSITY OF CAMBRIDGE INTERNATIONAL EXAMINATIONS …iteach.pk/wp-content/uploads/2015/02/computer-science-papers-A1-… · COMPUTING 9691/31 Paper 3 May/June 2011 2 hours ... 10

Page 10 Mark Scheme Syllabus Paper

GCE A LEVEL – May/June 2013 9691 32

© Cambridge International Examinations 2013

Question 8

(a) (i) Example [1]

(ii) two of the points explained ...

moveable … mechanical device ... sense its surroundings ... ... clear example // temperature, etc. controlled by a program ... [MAX 2]

(b) Robotic arm [1] Explained in the context of ‘their’ robot [1]

Sensor [1] Capture data [1] Actuator // Motor [1] To drive various motors to perform the robot’s movement [1] Microprocessor [1] To process the various inputs and execute the control program [1] Camera [1] To capture images [1] Memory [1] To temporarily store input data [1] Speaker [1] To provide audio output [1]

[MAX 4]

(c) real-time [1] [Total: 8]

M.Mushtaq Hussain

Contact the Teacher: 03215275281

Page 86: UNIVERSITY OF CAMBRIDGE INTERNATIONAL EXAMINATIONS …iteach.pk/wp-content/uploads/2015/02/computer-science-papers-A1-… · COMPUTING 9691/31 Paper 3 May/June 2011 2 hours ... 10

This document consists of 20 printed pages.

IB13 06_9691_33/4RP © UCLES 2013 [Turn over

*5582893707*

UNIVERSITY OF CAMBRIDGE INTERNATIONAL EXAMINATIONS

General Certificate of Education Advanced Level

COMPUTING 9691/33

Paper 3 May/June 2013

2 hours

Candidates answer on the Question Paper.

No additional materials are required.

No calculators allowed.

READ THESE INSTRUCTIONS FIRST

Write your Centre number, candidate number and name on all the work you hand in.

Write in dark blue or black pen.

You may use a soft pencil for any diagrams, graphs or rough working.

Do not use staples, paper clips, highlighters, glue or correction fluid.

DO NOT WRITE IN ANY BARCODES.

Answer all questions.

No marks will be awarded for using brand names for software packages or hardware.

At the end of the examination, fasten all your work securely together.

The number of marks is given in brackets [ ] at the end of each question or part question.

M.Mushtaq Hussain

Contact the Teacher: 03215275281

Page 87: UNIVERSITY OF CAMBRIDGE INTERNATIONAL EXAMINATIONS …iteach.pk/wp-content/uploads/2015/02/computer-science-papers-A1-… · COMPUTING 9691/31 Paper 3 May/June 2011 2 hours ... 10

2

© UCLES 2013 9691/33/M/J/13

For

Examiner's

Use

1 A database is created to store data about all the football clubs who play in a number of different leagues.

• Each club runs a number of different teams (Men, Women, Boys, Girls).

• Each club has a number of players.

• A player can only be registered with one club.

• Each club team plays in a league. Data is to be recorded in a relational database and the tables include CLUB and LEAGUE. (a) (i) What is the relationship between CLUB and LEAGUE?

[1]

(ii) Show this relationship with an entity-relationship (E-R) diagram. [1] (iii) Draw an E-R diagram showing a database design which can be produced so that

the club and league data are fully normalised. Explain how the relationships are implemented.

[4]

M.Mushtaq Hussain

Contact the Teacher: 03215275281

Page 88: UNIVERSITY OF CAMBRIDGE INTERNATIONAL EXAMINATIONS …iteach.pk/wp-content/uploads/2015/02/computer-science-papers-A1-… · COMPUTING 9691/31 Paper 3 May/June 2011 2 hours ... 10

3

© UCLES 2013 9691/33/M/J/13 [Turn over

For

Examiner's

Use

(b) (i) What is the relationship between CLUB and PLAYER?

[1]

(ii) Show this relationship with an E-R diagram. [1] (c) Two of the incomplete table designs are: CLUB(ClubName, GroundName, Address, ClubSecretaryName)

PLAYER(PlayerRegistrationNo, PlayerName, Gender, DateOfBirth,

PreferredPosition)

Explain how the relationship between CLUB and PLAYER is implemented.

[2]

(d) The following Data Manipulation Language query is run. SELECT PlayerRegistrationNo, PlayerName

FROM PLAYER

WHERE Gender=’F’ AND PreferredPosition=“Defender”

What useful information is produced from this query?

[2]

M.Mushtaq Hussain

Contact the Teacher: 03215275281

Page 89: UNIVERSITY OF CAMBRIDGE INTERNATIONAL EXAMINATIONS …iteach.pk/wp-content/uploads/2015/02/computer-science-papers-A1-… · COMPUTING 9691/31 Paper 3 May/June 2011 2 hours ... 10

6

© UCLES 2013 9691/33/M/J/13

For

Examiner's

Use

3 The table shows the assembly language instructions for a processor which has one general purpose register – the Accumulator (ACC), and an index register (IX).

Instruction

Op Code Operand Explanation

LDD <address> Direct addressing. Load the contents of the given address to ACC

STO <address> Store the contents of ACC at the given address

LDI <address> Indirect addressing. At the given address is the address to be used. Load the contents of this second address to ACC

LDX <address> Indexed addressing. Form the address as <address> +

the contents of IX. Copy the contents of this address to ACC

LIX <address> Load the contents of the given address to IX

INC <register> Add 1 to the contents of the register (ACC or IX)

ADD <address> Add the contents of the given address to the contents of ACC

OUT Output the contents of ACC (as a denary number) to the monitor

IN Input a denary number from the keyboard and store in ACC

JMP <address> Jump (unconditionally) to the given address

END End the program and return to the operating system

The diagrams on the next page show a program loaded in main memory starting at address

100. Two of the op-codes have been partially blanked out. Locations 200 onwards contain data which is used by the program.

M.Mushtaq Hussain

Contact the Teacher: 03215275281

Page 90: UNIVERSITY OF CAMBRIDGE INTERNATIONAL EXAMINATIONS …iteach.pk/wp-content/uploads/2015/02/computer-science-papers-A1-… · COMPUTING 9691/31 Paper 3 May/June 2011 2 hours ... 10

7

© UCLES 2013 9691/33/M/J/13 [Turn over

For

Examiner's

Use

(a) The instruction at address 100 is fetched and executed. Shown are the contents of the registers after execution.

100 LD■ 202

101 INC ACC

ACC 102 INC ACC

42 103 LD■ 203

104 INC ACC

105 LDD 204

IX 106 INC ACC

3 107 END

200 38

201 205

202 88

203 200

204 48

205 42

Which mode of addressing was used by this load instruction at address 100?

[1]

(b) The instruction at address 103 is fetched. Shown are the contents of the registers after

execution.

100 LD■ 202

101 INC ACC

ACC 102 INC ACC

38 103 LD■ 203

104 INC ACC

105 LDD 204

IX 106 INC ACC

3 107 END

200 38

201 205

202 88

203 200

204 48

205 42

Draw on the memory diagram to explain how this instruction works. Which mode of

addressing was used by this load instruction at address 103?

[2]

M.Mushtaq Hussain

Contact the Teacher: 03215275281

Page 91: UNIVERSITY OF CAMBRIDGE INTERNATIONAL EXAMINATIONS …iteach.pk/wp-content/uploads/2015/02/computer-science-papers-A1-… · COMPUTING 9691/31 Paper 3 May/June 2011 2 hours ... 10

8

© UCLES 2013 9691/33/M/J/13

For

Examiner's

Use

(c) Refer to the program used in (a) and (b). The instruction at address 105 is fetched and executed.

Show the contents of ACC after execution.

ACC

IX

[1] (d) Trace the first two iterations of this assembly language program using the trace table

below.

100 LIX 120

101 LDX 200

102 INC ACC

103 OUT

104 INC IX

105 JMP 101

120 0

200 165

201 93

202 107

ACC IX Output

[4]

M.Mushtaq Hussain

Contact the Teacher: 03215275281

Page 92: UNIVERSITY OF CAMBRIDGE INTERNATIONAL EXAMINATIONS …iteach.pk/wp-content/uploads/2015/02/computer-science-papers-A1-… · COMPUTING 9691/31 Paper 3 May/June 2011 2 hours ... 10

9

© UCLES 2013 9691/33/M/J/13 [Turn over

For

Examiner's

Use

(e) In (d) the program was shown in assembly language. In practice this must be machine code in order to execute the program.

Explain how the assembler software translates a program from assembly language into

machine code.

[3]

M.Mushtaq Hussain

Contact the Teacher: 03215275281

Page 93: UNIVERSITY OF CAMBRIDGE INTERNATIONAL EXAMINATIONS …iteach.pk/wp-content/uploads/2015/02/computer-science-papers-A1-… · COMPUTING 9691/31 Paper 3 May/June 2011 2 hours ... 10

14

© UCLES 2013 9691/33/M/J/13

For

Examiner's

Use

5 (a) Describe four differences between using a compiler or interpreter for the translation process and the execution of a high-level language source code program.

1

2

3

4

[4]

(b) The following are the first few lines of a source code program written using high-level

language XYZ which is about to be translated by the language compiler.

// program written 12 June 2013

Declare IsFound : Boolean;

Declare NoOfChildren : Integer;

Declare Count : Integer;

Constant TaxRate = 15;

// start of main program

For Count = 1 To 50

M.Mushtaq Hussain

Contact the Teacher: 03215275281

Page 94: UNIVERSITY OF CAMBRIDGE INTERNATIONAL EXAMINATIONS …iteach.pk/wp-content/uploads/2015/02/computer-science-papers-A1-… · COMPUTING 9691/31 Paper 3 May/June 2011 2 hours ... 10

15

© UCLES 2013 9691/33/M/J/13 [Turn over

For

Examiner's

Use

During the lexical analysis stage the compiler will use a keyword table and a symbol table.

(i) Describe what information is contained in these tables.

Keyword table

Symbol table

[3]

(ii) Explain how the table contents are used to translate the source code.

[2]

M.Mushtaq Hussain

Contact the Teacher: 03215275281

Page 95: UNIVERSITY OF CAMBRIDGE INTERNATIONAL EXAMINATIONS …iteach.pk/wp-content/uploads/2015/02/computer-science-papers-A1-… · COMPUTING 9691/31 Paper 3 May/June 2011 2 hours ... 10

19

© UCLES 2013 9691/33/M/J/13 [Turn over

For

Examiner's

Use

7 A user-defined function FoundBigger is defined, using pseudocode, as follows:

FUNCTION FoundBigger(ThisArray : INTEGER, UBound : INTEGER,

ThisValue : INTEGER) RETURNS BOOLEAN

The function checks each element in the array ThisArray with upper bound UBound.

The function returns a Boolean value to show if ThisValue is bigger than any of the values

in ThisArray.

If the function is incorrectly formed it will give a ‘COMPILE ERROR’.

The function is used with the three arrays shown below:

Subscript/Index

Identifier 1 2 3 4 5 6 7 8 9 10

List1 17 0 23 11 16 4

List2 13 16 16 0 20 22 20 19 11 23

List3 41 29 34 39 39 44 0

(a) What is returned by the following function calls? (i) FoundBigger(List3, 7, 50)

[1]

(ii) FoundBigger(List3, 7, 41)

[1]

(iii) FoundBigger(65, List1)

[1]

(iv) FoundBigger(List2, 10, “27”)

[1]

M.Mushtaq Hussain

Contact the Teacher: 03215275281

Page 96: UNIVERSITY OF CAMBRIDGE INTERNATIONAL EXAMINATIONS …iteach.pk/wp-content/uploads/2015/02/computer-science-papers-A1-… · COMPUTING 9691/31 Paper 3 May/June 2011 2 hours ... 10

20

Permission to reproduce items where third-party owned material protected by copyright is included has been sought and cleared where possible. Every reasonable effort has been made by the publisher (UCLES) to trace copyright holders, but if any items requiring clearance have unwittingly been included, the publisher will be pleased to make amends at the earliest possible opportunity.

University of Cambridge International Examinations is part of the Cambridge Assessment Group. Cambridge Assessment is the brand name of University of Cambridge Local Examinations Syndicate (UCLES), which is itself a department of the University of Cambridge.

© UCLES 2013 9691/33/M/J/13

For

Examiner's

Use

(b) A programmer writes pseudocode to calculate an employee’s net pay with a function CalcNetPay.

The calculation is done using:

• Employee’s pay grade (a single character: F, P or C)

• Hours worked that week Show the function header for CalcNetPay.

[3]

8 (a) Define what is meant by simulation.

[2]

(b) An application of simulation is used for producing accurate weather forecasts. Explain how the computer would carry out the simulation and why the use of a

computer system is appropriate.

[4]

M.Mushtaq Hussain

Contact the Teacher: 03215275281

Page 97: UNIVERSITY OF CAMBRIDGE INTERNATIONAL EXAMINATIONS …iteach.pk/wp-content/uploads/2015/02/computer-science-papers-A1-… · COMPUTING 9691/31 Paper 3 May/June 2011 2 hours ... 10

CAMBRIDGE INTERNATIONAL EXAMINATIONS

GCE Advanced Level

MARK SCHEME for the May/June 2013 series

9691 COMPUTING

9691/33 Paper 3 (Written Paper), maximum raw mark 90

This mark scheme is published as an aid to teachers and candidates, to indicate the requirements of the examination. It shows the basis on which Examiners were instructed to award marks. It does not indicate the details of the discussions that took place at an Examiners’ meeting before marking began, which would have considered the acceptability of alternative answers.

Mark schemes should be read in conjunction with the question paper and the Principal Examiner Report for Teachers. Cambridge will not enter into discussions about these mark schemes. Cambridge is publishing the mark schemes for the May/June 2013 series for most IGCSE, GCE Advanced Level and Advanced Subsidiary Level components and some Ordinary Level components.

M.Mushtaq Hussain

Contact the Teacher: 03215275281

Page 98: UNIVERSITY OF CAMBRIDGE INTERNATIONAL EXAMINATIONS …iteach.pk/wp-content/uploads/2015/02/computer-science-papers-A1-… · COMPUTING 9691/31 Paper 3 May/June 2011 2 hours ... 10

Page 2 Mark Scheme Syllabus Paper

GCE A LEVEL – May/June 2013 9691 33

© Cambridge International Examinations 2013

1 (a) (i) Many CLUBs play in many LEAGUEs. [1] (ii) E-R diagram [1]

(iii)

– Link table drawn [1] – 2 x one-to-many relationships [1] – primary key in CLUB links to foreign key in link table [1] – primary key in LEAGUE links to foreign key in link table [1] No mention of foreign keys scores max 1 for final mark points

(b) (i) One CLUB has many PLAYERs [1]

(ii) E-R diagram [1]

(c) The primary key of table CLUB – ClubName [1]

Matches to ClubName in the PLAYER table [1]

(d) Displays a ‘list’ of the player names and registration numbers [1] Who are female defenders [1]

[Total: 12] 2 (a) Meta language

Rules/Grammar (which describe a high level programming language // protocol specification)

The syntax or structure of all program statements [2]

(b) (i) A rule which is defined in terms of itself NB Not ‘procedure’ ….. [1]

(ii) Rule 6 [1] (iii)

M.Mushtaq Hussain

Contact the Teacher: 03215275281

Page 99: UNIVERSITY OF CAMBRIDGE INTERNATIONAL EXAMINATIONS …iteach.pk/wp-content/uploads/2015/02/computer-science-papers-A1-… · COMPUTING 9691/31 Paper 3 May/June 2011 2 hours ... 10

Page 4 Mark Scheme Syllabus Paper

GCE A LEVEL – May/June 2013 9691 33

© Cambridge International Examinations 2013

3 (a) Indexed addressing // LDX [1]

(b) Indirect addressing Annotation to explain that address 203 is used as a forwarding address [2]

(c) 48 [1]

(d)

ACC IX Output

0

165

166

166

1

93

94

94

(2)

Mark as follows: Index register contain 0 [1] Sequence of first box (or subsequent sequence for the same instructions) [1] Index register contains 1 [1] Sequence for final box [1] (e) Labels added to a (symbol) table // creates a list of addresses [1]

Labels are later looked up to determine the actual address / Assembler must allocate addresses to labels [1]

Mnemonic looked up to give binary code/machine code [1] Macro instructions are expanded into a group of instructions [1] The software makes two passes through the source program [1]

MAX 3

[Total: 11]

M.Mushtaq Hussain

Contact the Teacher: 03215275281

Page 100: UNIVERSITY OF CAMBRIDGE INTERNATIONAL EXAMINATIONS …iteach.pk/wp-content/uploads/2015/02/computer-science-papers-A1-… · COMPUTING 9691/31 Paper 3 May/June 2011 2 hours ... 10

Page 6 Mark Scheme Syllabus Paper

GCE A LEVEL – May/June 2013 9691 33

© Cambridge International Examinations 2013

(d) (i) ListTraversal(MyList[Index].Pointer) [1] (ii)

ListTraversal(3)

MyList[3].Pointer <> 0 is TRUE

ListTraversal(1)

MyList[1].Pointer <> 0 is TRUE

ListTraversal(2) MyList[2].Pointer <> 0 is TRUE ListTraversal(4)

MyList[4].Pointer <> 0 is FALSE

OUTPUT MyList[4].Data // OUTPUT PEPPER

ENDPROCEDURE

OUTPUT COURGETTE

ENDPROCEDURE

OUTPUT BEAN ENDPROCEDURE

OUTPUT APPLE ENDPROCEDURE

[4]

(iii) The procedure has to backtrack/unwind from the current call [1]

To return to the calling procedure // return to the addresses from which called [1]

MAX 1 [Total: 15]

5 (a) – Interpreter translates one instruction, runs it before going on to the next // Compiler translates all the instructions before run [1]

– Compiler creates object code/executable file // Interpreter does not [1] – Interpreter makes for easier debugging //compiler errors produced away from the

execution [1] – Compiled programs will execute faster // interpreted code will execute slower [1] – Interpreter must be present to run the program // compiler software not needed at

runtime [1] – Interpreter will translate code in loops more than once // Compiler only once [1] – Once compiled no further translation needed // Interpreter needed every program

execution attempted [1] MAX 4

M.Mushtaq Hussain

Contact the Teacher: 03215275281

Page 101: UNIVERSITY OF CAMBRIDGE INTERNATIONAL EXAMINATIONS …iteach.pk/wp-content/uploads/2015/02/computer-science-papers-A1-… · COMPUTING 9691/31 Paper 3 May/June 2011 2 hours ... 10

Page 7 Mark Scheme Syllabus Paper

GCE A LEVEL – May/June 2013 9691 33

© Cambridge International Examinations 2013

(b) (i) The keyword table contains: all the language keywords/reserved words + with a matching token [1] The symbol table stores: each identifier/variable found (and its data type) [1]

the values of all constants [1] the upper and lower bounds of arrays [1] Mark as: 1 + 1 MAX 2

(ii) Keywords are looked up in the keyword table [1]

Keywords are converted to tokens [1] Labels are looked up in the symbol table [1] Labels are converted to actual addresses [1]

MAX 2

[Total: 9]

6 (a) Batch is X [1] There is a time delay before processing [1] All the (data) is processed together/at the same time [1] The payslips are generated as a batch [1] Processing cannot start until all data has been collected/input // all data entered by the 18th

so processing can be done on the 25th [1] There is no user involvement [1]

MAX 2

(Interactive processing is Y) The user continually wants to see the effect of the changes/design produced [1] There is data input by the user [1]

MAX 1

(b) (i) STAFF17 can be loaded [1]

(ii) Partition 3 is too small [1]

It will not allow all 12 students to log-on at 09:00 [1]

(iii) Operating system // specific modules e.g. interrupt handler/scheduler, etc. [1]

device drivers [1]

examples of system software or utilities [1]

R. “System software” and “Utilities” MAX 2

(iv) Running [1]

The job currently has use of the processor [1]

Suspended/Blocked [1]

the program is unable use the processor/ or by example, the job is currently using an I/O device [1]

Note: the explanation marks are not dependant on the correct name

M.Mushtaq Hussain

Contact the Teacher: 03215275281

Page 102: UNIVERSITY OF CAMBRIDGE INTERNATIONAL EXAMINATIONS …iteach.pk/wp-content/uploads/2015/02/computer-science-papers-A1-… · COMPUTING 9691/31 Paper 3 May/June 2011 2 hours ... 10

Page 8 Mark Scheme Syllabus Paper

GCE A LEVEL – May/June 2013 9691 33

© Cambridge International Examinations 2013

(c) (i) The program is divided into fixed sized units called pages

The memory is divided into areas (with the same size) called page frames [1]

Page management is done using a ‘page frame table’ [1]

Pages will be swapped in and out of page frames as required. [1]

An area on the hard disc can act as virtual memory (to speed up the swapping) [1]

Virtual memory extends memory capacity / acts as pseudo-memory

MAX 2 (ii) Not all pages of the program need to loaded [1] saves memory [1] more jobs can be run [1] MAX 1 [Total: 16] 7 (a) (i) True / Yes [1] (ii) False / No [1] (iii) COMPILE ERROR [1] (iv) COMPILE ERROR [1] (b) FUNCTION CalcNetPay (EmpGrade:CHAR/STRING, HoursWorked:SINGLE/INTEGER) RETURN SINGLE

[1] [1]

[1]

A: RETURN REAL/CURRENCY

[Total: 7]

8 (a) Building a model of the system ..... [1]

The model records over time the result of changing parameters/conditions // Models the behaviour of the system [1]

(b) A computer program can be written to build the model [1] The computer system can process results very quickly / can reduce the time frame [1]

Weather forecasting has many changes which are based on mathematical equations [1] Inputs will originate from various sensors [1] e.g. wind speed / temperature / air pressure [1] powerful computers process results (from many sensor sources) [1] Can use parallel processing [1] Outputs will be produced which are based on all the available data [1] MAX 4 [Total: 6]

M.Mushtaq Hussain

Contact the Teacher: 03215275281

Page 103: UNIVERSITY OF CAMBRIDGE INTERNATIONAL EXAMINATIONS …iteach.pk/wp-content/uploads/2015/02/computer-science-papers-A1-… · COMPUTING 9691/31 Paper 3 May/June 2011 2 hours ... 10

This document consists of 16 printed pages.

IB13 11_9691_31/3RP © UCLES 2013

[Turn over

*9693515187*

UNIVERSITY OF CAMBRIDGE INTERNATIONAL EXAMINATIONS

General Certificate of Education Advanced Level

COMPUTING 9691/31

Paper 3 October/November 2013

2 hours

Candidates answer on the Question Paper.

No additional materials are required.

No calculators allowed.

READ THESE INSTRUCTIONS FIRST

Write your Centre number, candidate number and name on all the work you hand in.

Write in dark blue or black pen.

You may use a soft pencil for any diagrams, graphs or rough working.

Do not use staples, paper clips, highlighters, glue or correction fluid.

DO NOT WRITE IN ANY BARCODES.

Answer all questions.

No marks will be awarded for using brand names for software packages or hardware.

At the end of the examination, fasten all your work securely together.

The number of marks is given in brackets [ ] at the end of each question or part question.

M.Mushtaq Hussain

Contact the Teacher: 03215275281

Page 104: UNIVERSITY OF CAMBRIDGE INTERNATIONAL EXAMINATIONS …iteach.pk/wp-content/uploads/2015/02/computer-science-papers-A1-… · COMPUTING 9691/31 Paper 3 May/June 2011 2 hours ... 10

4

© UCLES 2013 9691/31/O/N/13

For

Examiner's

Use

2 Cross country runners take part in races.

• A runner must be registered with one club only and club names are unique.

• A club has runners; each runner has a unique national MemberID.

• Each race is organised by a club and the Club Secretary records which runners are

entered for each race.

• Runners may enter any race.

• There is only one race on any one day.

At present each club records the data for the competition races it organises. The data is

stored in flat files.

(a) Describe three advantages that a relational database would have over the use of flat

files.

1

2

3

[3]

(b) (i) What is the relationship between runner and race?

[1]

(ii) What is the relationship between club and race?

[1]

(c) A database solution is to be developed.

Two of the tables are RUNNER and RACE.

(i) Draw an entity-relationship (E-R) diagram showing a database design which can

be produced so that the runner and race data are fully normalised.

[2]

M.Mushtaq Hussain

Contact the Teacher: 03215275281

Page 105: UNIVERSITY OF CAMBRIDGE INTERNATIONAL EXAMINATIONS …iteach.pk/wp-content/uploads/2015/02/computer-science-papers-A1-… · COMPUTING 9691/31 Paper 3 May/June 2011 2 hours ... 10

5

© UCLES 2013 9691/31/O/N/13 [Turn over

For

Examiner's

Use

(ii) Explain how the relationships are implemented.

[2]

(d) The following table design is suggested for RUNNER.

RUNNER(MemberID, RunnerName, RunnerDOB, ClubName, ClubAddress)

This is poorly designed.

(i) Is this table in First Normal Form (1NF)?

Explain.

[1]

(ii) Is this table in Second Normal form (2NF)?

Explain.

[1]

(iii) The table is not in Third Normal Form (3NF).

Explain.

[1]

(iv) Using only the attributes given in the RUNNER table above, produce a new design

which is fully normalised.

The table descriptions should be expressed as:

TableName(Attribute1, Attribute2, Attribute3, ...)

[2]

M.Mushtaq Hussain

Contact the Teacher: 03215275281

Page 106: UNIVERSITY OF CAMBRIDGE INTERNATIONAL EXAMINATIONS …iteach.pk/wp-content/uploads/2015/02/computer-science-papers-A1-… · COMPUTING 9691/31 Paper 3 May/June 2011 2 hours ... 10

6

© UCLES 2013 9691/31/O/N/13

For

Examiner's

Use

(e) Explain why all tables in the final design should be fully normalised.

[2]

(f) The table to store the race data has the following design:

RACE(RaceDate, RaceStartTime, StartVenue, Distance,

OrganisingClubName)

Write a Data Manipulation Language (DML) query to report all races after the 1st

January 2013 which are less than 10 km. Display the race date and organising club

name only.

Use the keywords SELECT, FROM, WHERE.

[3]

3 (a) Most modern computers are designed using Von Neumann architecture.

Explain what is meant by Von Neumann architecture.

[2]

(b) (i) Convert the hexadecimal number 7A to denary.

[1]

(ii) Convert the binary number 0101 1100 to hexadecimal.

[1]

M.Mushtaq Hussain

Contact the Teacher: 03215275281

Page 107: UNIVERSITY OF CAMBRIDGE INTERNATIONAL EXAMINATIONS …iteach.pk/wp-content/uploads/2015/02/computer-science-papers-A1-… · COMPUTING 9691/31 Paper 3 May/June 2011 2 hours ... 10

7

© UCLES 2013 9691/31/O/N/13 [Turn over

For

Examiner's

Use

(iii) Why do computer scientists often write binary numbers in hexadecimal?

[1]

(c) The diagram shows a program loaded into main memory starting at memory address

7A Hex.

Address

Main memory

(contents shown in

Hex.)

7A

2150

7B

A351

7C

A552

7D

FFFF

90

003C

(i) How many bits are used for each main memory location?

[1]

The trace table below is used to show how the contents of the special-purpose

registers change as the program is executed.

The steps in the fetch stage of the fetch-execute cycle are shown in the first column

using register transfer notation. (For example, MAR ← [PC] means the content of the

Program Counter is copied to the Memory Address Register.)

(ii) Complete the trace table for the fetching of the first program instruction (2150):

• Show the changing contents of the registers

• Put a tick in the Address bus/Data bus column to show when the signals on

that bus change.

Special purpose registers (Contents shown in Hex.)

Buses Fetch stage

PC MAR MDR CIR Address bus Data bus

7A

MAR ← [PC]

PC ← [PC] + 1

MDR ← [[MAR]]

CIR ← [MDR]

[5]

M.Mushtaq Hussain

Contact the Teacher: 03215275281

Page 108: UNIVERSITY OF CAMBRIDGE INTERNATIONAL EXAMINATIONS …iteach.pk/wp-content/uploads/2015/02/computer-science-papers-A1-… · COMPUTING 9691/31 Paper 3 May/June 2011 2 hours ... 10

8

© UCLES 2013 9691/31/O/N/13

For

Examiner's

Use

(d) The following table shows some of a processor’s instruction set in assembly language.

Instruction

Op Code Operand

Explanation

LDD <address> Direct addressing. Load the contents of the given address

to ACC

LDI <address> Indirect addressing. At the given address is the address to

be used. Load the contents of this second address to ACC

LIX <address> Load the contents of the address to the Index register (IX)

LDX <address> Indexed addressing. Form the address as <address> +

the contents of IX. Copy the contents of this address to ACC

The following program is to be executed. Shown are:

• the first four instructions only of this program

• the memory locations which are accessed by this program.

Address Main memory

100

LIX 200

101

LDD 201

102

LDI 201

103

LDX 201

200

3

201

216

202

99

203

217

204

63

216

96

217

97

Complete the trace table below for the first four program instructions.

Show each change in the contents of the registers.

Register

Instruction

Accumulator (ACC) Index Register (IX)

LIX 200

LDD 201

LDI 201

LDX 201

[4]

M.Mushtaq Hussain

Contact the Teacher: 03215275281

Page 109: UNIVERSITY OF CAMBRIDGE INTERNATIONAL EXAMINATIONS …iteach.pk/wp-content/uploads/2015/02/computer-science-papers-A1-… · COMPUTING 9691/31 Paper 3 May/June 2011 2 hours ... 10

15

© UCLES 2013 9691/31/O/N/13 [Turn over

For

Examiner's

Use

(b) (i) The processor receives an interrupt. This triggers the following sequence of steps.

1. Save the contents of the Program Counter on the

2. Also save

3. Load and run the appropriate

4. Restore what was saved at step 2

5. Restore the

6. Continue execution of the interrupted process

Complete the statements above. [4]

(ii) Interrupts can be allocated priorities.

While execution is occurring at step 3, a higher priority interrupt is received.

Explain what additional steps must now be added to the sequence in (b)(i). State

where in the sequence these additions occur.

[3]

7 Encryption of data is widely used in computing.

(a) One application is the sending of payment data using a debit/credit card for an online

purchase.

State two other applications where encryption is used.

Describe the reason for encrypting the data for each application.

Application 1

Reason

Application 2

Reason

[4]

M.Mushtaq Hussain

Contact the Teacher: 03215275281

Page 110: UNIVERSITY OF CAMBRIDGE INTERNATIONAL EXAMINATIONS …iteach.pk/wp-content/uploads/2015/02/computer-science-papers-A1-… · COMPUTING 9691/31 Paper 3 May/June 2011 2 hours ... 10

16

Permission to reproduce items where third party owned material protected by copyright is included has been sought and cleared where possible. Every reasonable effort has been made by the publisher (UCLES) to trace copyright holders, but if any items requiring clearance have unwittingly been included, the publisher will be pleased to make amends at the earliest possible opportunity.

University of Cambridge International Examinations is part of the Cambridge Assessment Group. Cambridge Assessment is the brand name of University of Cambridge Local Examinations Syndicate (UCLES), which is itself a department of the University of Cambridge.

© UCLES 2013 9691/31/O/N/13

For

Examiner's

Use

(b) Explain the terms encryption algorithm and encryption key.

Encryption algorithm

Encryption key

[2]

(c) Asymmetric encryption uses both a public key and a private key.

Explain how they work together to encrypt and decrypt a message.

[3]

(d) Authorisation and authentication are processes designed to protect the computer

system and data.

Give one technique used for each.

Authorisation

Authentication

[2]

M.Mushtaq Hussain

Contact the Teacher: 03215275281

Page 111: UNIVERSITY OF CAMBRIDGE INTERNATIONAL EXAMINATIONS …iteach.pk/wp-content/uploads/2015/02/computer-science-papers-A1-… · COMPUTING 9691/31 Paper 3 May/June 2011 2 hours ... 10

This document consists of 19 printed pages and 1 blank page.

IB13 11_9691_32/7RP © UCLES 2013 [Turn over

*2685241834*

UNIVERSITY OF CAMBRIDGE INTERNATIONAL EXAMINATIONS

General Certificate of Education Advanced Level

COMPUTING 9691/32

Paper 3 October/November 2013

2 hours

Candidates answer on the Question Paper.

No additional materials are required.

No calculators allowed.

READ THESE INSTRUCTIONS FIRST

Write your Centre number, candidate number and name on all the work you hand in.

Write in dark blue or black pen.

You may use a soft pencil for any diagrams, graphs or rough working.

Do not use staples, paper clips, highlighters, glue or correction fluid.

DO NOT WRITE IN ANY BARCODES.

Answer all questions.

No marks will be awarded for using brand names for software packages or hardware.

At the end of the examination, fasten all your work securely together.

The number of marks is given in brackets [ ] at the end of each question or part question.

M.Mushtaq Hussain

Contact the Teacher: 03215275281

Page 112: UNIVERSITY OF CAMBRIDGE INTERNATIONAL EXAMINATIONS …iteach.pk/wp-content/uploads/2015/02/computer-science-papers-A1-… · COMPUTING 9691/31 Paper 3 May/June 2011 2 hours ... 10

4

© UCLES 2013 9691/32/O/N/13

For

Examiner's

Use

2 Customers order products from a website.

• An order contains one or more products.

• Over time, a customer places many orders.

• A product will appear on many customer orders.

• Each product is sourced from a single supplier and a number of suppliers are used.

• A supplier can supply more than one product. At present the company stores and manages all the data using flat files. (a) Describe three advantages that a relational database would have over the use of flat

files.

1

2

3

[3]

(b) (i) What is the relationship between product and supplier?

[1]

(ii) What is the relationship between product and order?

[1]

(c) A database solution is to be developed. Two of the tables are PRODUCT and ORDER.

(i) Draw an entity-relationship (E-R) diagram showing a database design which can

be produced so that the product and order data are fully normalised. [2]

M.Mushtaq Hussain

Contact the Teacher: 03215275281

Page 113: UNIVERSITY OF CAMBRIDGE INTERNATIONAL EXAMINATIONS …iteach.pk/wp-content/uploads/2015/02/computer-science-papers-A1-… · COMPUTING 9691/31 Paper 3 May/June 2011 2 hours ... 10

5

© UCLES 2013 9691/32/O/N/13 [Turn over

For

Examiner's

Use

(ii) Explain how the relationships are implemented.

[2]

(d) The following table design is suggested for PRODUCT.

PRODUCT(ProductID, ProductDescription, RetailPrice, SupplierID,

SupplierName, SupplierTelNumber)

This is poorly designed. (i) Is this table in First Norm Form (1NF)? Explain.

[1]

(ii) Is this table in Second Normal form (2NF)? Explain.

[1]

(iii) The table is not in Third Normal Form (3NF). Explain.

[1]

(iv) Using only the attributes given in the PRODUCT table above, produce a new design

which is fully normalised. The table descriptions should be expressed as: TableName(Attribute1, Attribute2, Attribute3, ...)

[2]

M.Mushtaq Hussain

Contact the Teacher: 03215275281

Page 114: UNIVERSITY OF CAMBRIDGE INTERNATIONAL EXAMINATIONS …iteach.pk/wp-content/uploads/2015/02/computer-science-papers-A1-… · COMPUTING 9691/31 Paper 3 May/June 2011 2 hours ... 10

6

© UCLES 2013 9691/32/O/N/13

For

Examiner's

Use

(e) Explain why all tables in the final design should be fully normalised.

[2]

(f) The table to store the order data has the following design: ORDER(OrderNo, OrderDate, OrderTime, IsPaid, OrderAmountPaid,

PaymentMethod, CustomerID)

• IsPaid has data type Boolean

• PaymentMethod has data type Char with possible values: C - credit card,

D - debit card, A - account customer Write a Data Manipulation Language (DML) query to report orders which were placed

on the 15 January 2013 and paid for using a debit card. Show the customer ID and order number only.

Use the keywords SELECT, FROM, WHERE.

[3]

3 (a) Describe what is meant by a register.

[2]

(b) (i) Convert the denary number 60 into hexadecimal.

[1]

(ii) Convert the hexadecimal number 10F into denary.

[1]

M.Mushtaq Hussain

Contact the Teacher: 03215275281

Page 115: UNIVERSITY OF CAMBRIDGE INTERNATIONAL EXAMINATIONS …iteach.pk/wp-content/uploads/2015/02/computer-science-papers-A1-… · COMPUTING 9691/31 Paper 3 May/June 2011 2 hours ... 10

7

© UCLES 2013 9691/32/O/N/13 [Turn over

For

Examiner's

Use

(iii) Why do computer scientists often write binary numbers in hexadecimal?

[1]

(c) The diagram shows a program loaded into main memory starting at memory address

30 Hex.

Address

Main memory (contents shown in

Hex.)

30 2150

31 A351

32 A552

33 FFFF

58 003C

59 103C

5A 010B

(i) How many bytes are used to store each program instruction?

[1]

(ii) Describe the steps in the fetch stage of the fetch-execute cycle. Refer to the instruction at address 30 to illustrate your answer.

[5]

M.Mushtaq Hussain

Contact the Teacher: 03215275281

Page 116: UNIVERSITY OF CAMBRIDGE INTERNATIONAL EXAMINATIONS …iteach.pk/wp-content/uploads/2015/02/computer-science-papers-A1-… · COMPUTING 9691/31 Paper 3 May/June 2011 2 hours ... 10

8

© UCLES 2013 9691/32/O/N/13

For

Examiner's

Use

(d) The following table shows some of a processor’s instruction set in assembly language.

Instruction

Op Code Operand Explanation

LIX <address> Load the contents of the address to the Index register (IX)

LDX <address> Indexed addressing. Form the address as <address> +

the contents of IX. Copy the contents of this address to ACC

STO <address> Store the contents of ACC at the given address

ADD <address> Add the contents of the given address to the ACC

INC <register> Add 1 to the contents of the register (ACC or IX)

JMP <address> Jump to the given address

The following program is to be executed. Shown are:

• the first six instructions of this program

• the memory locations which will be accessed by this program.

Address Main memory

contents

100 LIX 200

101 LDX 200

102 ADD 204

103 STO 204

104 INC IX

105 JMP 101

200 1

201 13

202 14

203 22

204 0

M.Mushtaq Hussain

Contact the Teacher: 03215275281

Page 117: UNIVERSITY OF CAMBRIDGE INTERNATIONAL EXAMINATIONS …iteach.pk/wp-content/uploads/2015/02/computer-science-papers-A1-… · COMPUTING 9691/31 Paper 3 May/June 2011 2 hours ... 10

9

© UCLES 2013 9691/32/O/N/13 [Turn over

For

Examiner's

Use

Complete the trace table below for three iterations of the loop. Show each change to the contents of the registers and memory location 204.

ACC IX Main memory address 204

0

[4] 4 Object-oriented programming is one programming paradigm. (a) Explain the difference between a class and an object.

[3]

M.Mushtaq Hussain

Contact the Teacher: 03215275281

Page 118: UNIVERSITY OF CAMBRIDGE INTERNATIONAL EXAMINATIONS …iteach.pk/wp-content/uploads/2015/02/computer-science-papers-A1-… · COMPUTING 9691/31 Paper 3 May/June 2011 2 hours ... 10

16

© UCLES 2013 9691/32/O/N/13

For

Examiner's

Use

6 (a) The integers -126 and -5 are to be added. Write the binary for -126 and -5 using two’s complement. Show the addition in binary

including any carry bits.

-126

-5 +

Comment on the answer.

[4]

(b) Real numbers are to be stored using floating point representation with:

• 8 bits for the mantissa, followed by,

• 4 bits for the exponent

• two’s-complement is used for both the mantissa and exponent (i) Consider the binary pattern:

0 1 1 0 1 0 0 0 0 0 1 1

What number is this in denary? Show your working.

[3]

(ii) Explain how you can recognise that the above number is normalised.

[1]

M.Mushtaq Hussain

Contact the Teacher: 03215275281

Page 119: UNIVERSITY OF CAMBRIDGE INTERNATIONAL EXAMINATIONS …iteach.pk/wp-content/uploads/2015/02/computer-science-papers-A1-… · COMPUTING 9691/31 Paper 3 May/June 2011 2 hours ... 10

18

© UCLES 2013 9691/32/O/N/13

For

Examiner's

Use

7 The encryption of data is widely used in computing. (a) One application is online banking. State two other applications where encryption is used. Describe the reason for encrypting the data for each application.

Application 1

Reason

Application 2

Reason

[4]

(b) Explain the terms plain text and cipher text.

Plain text

Cipher text

[2]

(c) Symmetric encryption uses a single key. Explain how a message is encrypted and decrypted using symmetric encryption.

[3]

M.Mushtaq Hussain

Contact the Teacher: 03215275281

Page 120: UNIVERSITY OF CAMBRIDGE INTERNATIONAL EXAMINATIONS …iteach.pk/wp-content/uploads/2015/02/computer-science-papers-A1-… · COMPUTING 9691/31 Paper 3 May/June 2011 2 hours ... 10

19

© UCLES 2013 9691/32/O/N/13

For

Examiner's

Use

(d) Authorisation and authentication are processes designed to protect the computer system and data.

Give one technique used for each.

Authorisation

Authentication

[2]

M.Mushtaq Hussain

Contact the Teacher: 03215275281

Page 121: UNIVERSITY OF CAMBRIDGE INTERNATIONAL EXAMINATIONS …iteach.pk/wp-content/uploads/2015/02/computer-science-papers-A1-… · COMPUTING 9691/31 Paper 3 May/June 2011 2 hours ... 10

This document consists of 18 printed pages and 2 blank pages.

IB13 11_9691_33/4RP © UCLES 2013 [Turn over

*9188780506*

UNIVERSITY OF CAMBRIDGE INTERNATIONAL EXAMINATIONS

General Certificate of Education Advanced Level

COMPUTING 9691/33

Paper 3 October/November 2013

2 hours

Candidates answer on the Question Paper.

No additional materials are required.

No calculators allowed.

READ THESE INSTRUCTIONS FIRST

Write your Centre number, candidate number and name on all the work you hand in.

Write in dark blue or black pen.

You may use a soft pencil for any diagrams, graphs or rough working.

Do not use staples, paper clips, highlighters, glue or correction fluid.

DO NOT WRITE IN ANY BARCODES.

Answer all questions.

No marks will be awarded for using brand names for software packages or hardware.

At the end of the examination, fasten all your work securely together.

The number of marks is given in brackets [ ] at the end of each question or part question.

M.Mushtaq Hussain

Contact the Teacher: 03215275281

Page 122: UNIVERSITY OF CAMBRIDGE INTERNATIONAL EXAMINATIONS …iteach.pk/wp-content/uploads/2015/02/computer-science-papers-A1-… · COMPUTING 9691/31 Paper 3 May/June 2011 2 hours ... 10

4

© UCLES 2013 9691/33/O/N/13

For

Examiner's

Use

2 A car hire company in a large town hires out cars to customers.

• There are five depots.

• A number of cars are based at each depot.

• Each car registration number is unique.

• Each customer hire is for a single car only.

• Customers may return for future car hires.

• A customer’s future hire may involve a different car. At present the company records all car, customer and hire data in flat files. (a) Describe three advantages that a relational database would have over the use of flat

files.

1

2

3

[3]

(b) (i) What is the relationship between car and customer?

[1]

(ii) What is the relationship between depot and car?

[1]

(c) A database solution is to be developed. Two of the tables are CAR and CUSTOMER.

(i) Draw an entity-relationship (E-R) diagram showing a database design which can

be produced so that the car and customer data are fully normalised. [2]

M.Mushtaq Hussain

Contact the Teacher: 03215275281

Page 123: UNIVERSITY OF CAMBRIDGE INTERNATIONAL EXAMINATIONS …iteach.pk/wp-content/uploads/2015/02/computer-science-papers-A1-… · COMPUTING 9691/31 Paper 3 May/June 2011 2 hours ... 10

5

© UCLES 2013 9691/33/O/N/13 [Turn over

For

Examiner's

Use

(ii) Explain how the relationships are implemented.

[2]

(d) The following table design is suggested for CAR.

CAR(CarRegistrationNo, CarMake, CarModel, HirePriceCode, DepotID,

DepotAddress, DepotManager)

This is poorly designed. (i) Is this table in First Norm Form (1NF)? Explain.

[1]

(ii) Is this table in Second Normal form (2NF)? Explain.

[1]

(iii) The table is not in Third Normal Form (3NF). Explain.

[1]

(iv) Using only the attributes given in the CAR table above, produce a new design

which is fully normalised. The table descriptions should be expressed as: TableName(Attribute1, Attribute2, Attribute3, ...)

[2]

M.Mushtaq Hussain

Contact the Teacher: 03215275281

Page 124: UNIVERSITY OF CAMBRIDGE INTERNATIONAL EXAMINATIONS …iteach.pk/wp-content/uploads/2015/02/computer-science-papers-A1-… · COMPUTING 9691/31 Paper 3 May/June 2011 2 hours ... 10

6

© UCLES 2013 9691/33/O/N/13

For

Examiner's

Use

(e) Explain why all tables in the final design should be fully normalised.

[2]

(f) The table to store the hire data has the following design: HIRE(HireID, CarRegistrationNo, HireBookingDate, HireStartDate,

NoOfDays, HireRate, CustomerID)

Write a Data Manipulation Language (DML) query to report all hire bookings made for

car registration 456431 with customer C674. Display the customer ID and hire ID only. Use the keywords SELECT, FROM, WHERE.

[3]

3 (a) Describe what is meant by a register.

[2]

(b) (i) Convert the hexadecimal number 7F into denary.

[1]

(ii) Convert the denary number 291 into hexadecimal.

[1]

(iii) Why do computer scientists often write binary numbers in hexadecimal?

[1]

M.Mushtaq Hussain

Contact the Teacher: 03215275281

Page 125: UNIVERSITY OF CAMBRIDGE INTERNATIONAL EXAMINATIONS …iteach.pk/wp-content/uploads/2015/02/computer-science-papers-A1-… · COMPUTING 9691/31 Paper 3 May/June 2011 2 hours ... 10

7

© UCLES 2013 9691/33/O/N/13 [Turn over

For

Examiner's

Use

(c) The diagram shows a program loaded into main memory starting at memory address 40 Hex.

Address

Main memory (Contents shown in

Hex.)

40 7324

41 A351

42 A552

43 FFFF

68 003C

69 103C

6A 010B

(i) How many bytes are used to store each program instruction?

[1]

(ii) Describe the steps in the fetch stage of the fetch-execute cycle. Use the instruction at address 40 to illustrate your answer.

[5]

M.Mushtaq Hussain

Contact the Teacher: 03215275281

Page 126: UNIVERSITY OF CAMBRIDGE INTERNATIONAL EXAMINATIONS …iteach.pk/wp-content/uploads/2015/02/computer-science-papers-A1-… · COMPUTING 9691/31 Paper 3 May/June 2011 2 hours ... 10

8

© UCLES 2013 9691/33/O/N/13

For

Examiner's

Use

(d) The following table shows some of a processor’s instruction set in assembly language.

Instruction

Op Code Operand Explanation

LDD <address> Direct addressing. Load the contents of the given address to ACC

LDI <address> Indirect addressing. At the given address is the address to be used. Load the contents of this second address to ACC

STO <address> Store the contents of ACC at the given address

ADD <address> Add the contents of the given address to the ACC

INC <register> Add 1 to the contents of the register (ACC or IX)

JMP <address> Jump to the given address

The following program is to be executed. Shown are:

• the first seven instructions in this program

• the memory locations which will be accessed by this program.

Address Main memory

130 LDI 160

131 ADD 153

132 STO 153

133 LDD 160

134 INC ACC

135 STO 160

136 JMP 130

150 13

151 23

152 11

153 0

160 150

M.Mushtaq Hussain

Contact the Teacher: 03215275281

Page 127: UNIVERSITY OF CAMBRIDGE INTERNATIONAL EXAMINATIONS …iteach.pk/wp-content/uploads/2015/02/computer-science-papers-A1-… · COMPUTING 9691/31 Paper 3 May/June 2011 2 hours ... 10

9

© UCLES 2013 9691/33/O/N/13 [Turn over

For

Examiner's

Use

Complete the trace table below for two iterations of the loop. Show each change in the contents of the register and memory locations.

Register Memory location ACC 153 160

0 150

[4]

M.Mushtaq Hussain

Contact the Teacher: 03215275281

Page 128: UNIVERSITY OF CAMBRIDGE INTERNATIONAL EXAMINATIONS …iteach.pk/wp-content/uploads/2015/02/computer-science-papers-A1-… · COMPUTING 9691/31 Paper 3 May/June 2011 2 hours ... 10

16

© UCLES 2013 9691/33/O/N/13

For

Examiner's

Use

6 (a) State where the computer’s boot file would be stored. Explain how the boot file is used to make the computer system ready for use.

[3]

(b) (i) Explain what is meant by an interrupt.

[1]

(ii) An example of an interrupt generated by a hardware device is the process in which

a printer signals that it is out of paper. Give two further examples of interrupts, one which is hardware generated, and

one which is generated by an executing program.

Hardware generated

Program generated

[2]

(c) In a multiprogramming environment several processes are concurrently loaded into

main memory. Each process is in one of three states: RUNNING, READY, SUSPENDED.

Explain these three terms.

RUNNING

READY

SUSPENDED

[3]

M.Mushtaq Hussain

Contact the Teacher: 03215275281

Page 129: UNIVERSITY OF CAMBRIDGE INTERNATIONAL EXAMINATIONS …iteach.pk/wp-content/uploads/2015/02/computer-science-papers-A1-… · COMPUTING 9691/31 Paper 3 May/June 2011 2 hours ... 10

17

© UCLES 2013 9691/33/O/N/13 [Turn over

For

Examiner's

Use

7 (a) Below are some terms and definitions for devices used for networking. (i) Match up each device on the left with its definition. Draw a line connecting each

description to the appropriate network device. (ii) Complete the missing component name.

Router Hardware or software to control unauthorised access to a private network

Bridge Hardware used to convert analogue signals to digital signals (and vice versa)

Firewall Hardware used to connect nodes in a circuit switching network

Switch Circuit board which connects the computer to a network

Modem Device to direct packets across a packet switched network

Device used to connect two bus network segments to allow communication between all nodes

[6]

M.Mushtaq Hussain

Contact the Teacher: 03215275281

Page 130: UNIVERSITY OF CAMBRIDGE INTERNATIONAL EXAMINATIONS …iteach.pk/wp-content/uploads/2015/02/computer-science-papers-A1-… · COMPUTING 9691/31 Paper 3 May/June 2011 2 hours ... 10

18

© UCLES 2013 9691/33/O/N/13

For

Examiner's

Use

(b) (i) Networks use a variety of different media for communication. Name and describe two of these media.

Medium 1

Medium 2

[4]

(ii) A new communication link is to be constructed in a network. Name one factor that will be considered when deciding on the medium to be used.

[1]

M.Mushtaq Hussain

Contact the Teacher: 03215275281

Page 131: UNIVERSITY OF CAMBRIDGE INTERNATIONAL EXAMINATIONS …iteach.pk/wp-content/uploads/2015/02/computer-science-papers-A1-… · COMPUTING 9691/31 Paper 3 May/June 2011 2 hours ... 10

Page 2 Mark Scheme Syllabus Paper

GCE A LEVEL – October/November 2013 9691 31

© Cambridge International Examinations 2013

1 (a) (i) a b + 7 / [1] (ii) 2 3 z * 5 + / 1 [1] 2nd mark for completely correct [1] (b) evidence for 12 and 4 [1] 3 [1] (c) (i) In-order traversal // (Traverse each subtree in the order) left-root-right [1] (ii) E M c 2 ^ * = [1] (iii) Post-order traversal // (Traverse each subtree in the order) left-right-root [1] [Total: 8] 2 (a) Security is improved/better managed [1]

Different users can have different ‘views’ of/access to data [1] Program-data independence // Changing a field does not require an applications program re-write [1] Queries and reports quickly produced [1] Reduced data duplication/redundancy [1] Reduced data inconsistencies [1] Better managed data integrity/data validation // Validation code does not need to be present in all applications programs [1] If implemented with a DBMS it will allow concurrent access to the database [1] MAX 3

(b) (i) many runners compete in many races // many-to-many // M:m [1] (ii) one club organises many races // one-to-many // 1:M [1] (c) (i)

Intermediate table (not labelled RUNNER, RACE, CLUB, etc.) [1] 2 X one-to-many relationship [1]

(ii) Primary key of RACE/Primary key RaceDate // Primary key of RUNNER/Primary key MemberID [1] Is used as a foreign key in the link table [1] (d) (i) (Yes) since there is a not a repeated group of attributes [1] (ii) (Yes) Since there is only a single attribute primary key

// there are no partial dependencies // all non-key attr. are dependent on the primary key [1]

M.Mushtaq Hussain

Contact the Teacher: 03215275281

Page 132: UNIVERSITY OF CAMBRIDGE INTERNATIONAL EXAMINATIONS …iteach.pk/wp-content/uploads/2015/02/computer-science-papers-A1-… · COMPUTING 9691/31 Paper 3 May/June 2011 2 hours ... 10

Page 3 Mark Scheme Syllabus Paper

GCE A LEVEL – October/November 2013 9691 31

© Cambridge International Examinations 2013

(iii) There are dependent non-key attributes // ClubAddress is dependant on ClubName [1]

(iv) RUNNER(MemberID, RunnerName, RunnerDOB, ClubName) [1]

CLUB(ClubName, ClubAddress) [1] If primary key not indicated penalise once only

(e) Avoids data duplication/repeated data [1] Avoids data inconsistencies [1] Ensures data integrity [1]

(f) SELECT RaceDate, OrganisingClubName [1] FROM RACE [1] WHERE RaceDate > #01/01/2013# AND Distance < 10 [1]

Do not penalise imprecise syntax in the WHERE line

[Total: 19] 3 (a) a single processor

program consists of a sequence of stored instructions [1] Instructions + data [1] are stored (in a continuous block) of primary/main memory [1] instructions are executed in sequence [1]

MAX 2

(b) (i) 122 [1] (ii) 5C [1] (iii) Fewer digits used to represent any number // long string difficult to interpret [1]

Less likely to make a mistake when copying/converting a digit string [1] Easy to convert from binary to hex (vice versa) than binary to denary [1]

MAX 1

M.Mushtaq Hussain

Contact the Teacher: 03215275281

Page 133: UNIVERSITY OF CAMBRIDGE INTERNATIONAL EXAMINATIONS …iteach.pk/wp-content/uploads/2015/02/computer-science-papers-A1-… · COMPUTING 9691/31 Paper 3 May/June 2011 2 hours ... 10

Page 4 Mark Scheme Syllabus Paper

GCE A LEVEL – October/November 2013 9691 31

© Cambridge International Examinations 2013

(c) (i) 16 bits [1]

(ii)

Fetch stages

Special purpose registers Busses

PC MAR MDR CIR

Address bus

Data bus

7A

MAR ← [PC] 7A

PC ← [PC] + 1 7B

MDR ←[[MAR]] 2150

CIR ← [MDR] 2150

For the buses column penalise once for any additional incorrect ticks MAX 5 (d)

Register Instruction Accumulator

(ACC) Index Register

(IX)

LIX 200 3

LDD 201 216

LDI 201 96

LDX 201 63

1 per contents [4]

[Total: 15] 4 A class is the design/blueprint/template (from which objects are later created) [1]

A class consists of properties/attributes and methods/procedures/functions [1] An object is an instance of a class [1] An object must be based on a class definition [1] Many objects can exist for the same class [1]

MAX 3

M.Mushtaq Hussain

Contact the Teacher: 03215275281

Page 134: UNIVERSITY OF CAMBRIDGE INTERNATIONAL EXAMINATIONS …iteach.pk/wp-content/uploads/2015/02/computer-science-papers-A1-… · COMPUTING 9691/31 Paper 3 May/June 2011 2 hours ... 10

Page 5 Mark Scheme Syllabus Paper

GCE A LEVEL – October/November 2013 9691 31

© Cambridge International Examinations 2013

(b) The class diagram includes: BOOK + RECORDING subclasses [1] FILM + MUSIC subclasses of RECORDING [1] Recognised notation for inheritance [1] RESOURCE class Title : STRING

OnLoan : BOOLEAN [1]

BOOK class Author : STRING [1] FILM class RunningTime : INTEGER [1] MUSIC class NoOfTracks : INTEGER [1] RECORDING class ReleaseDate : DATE [1]

MAX 8 (c) Encapsulation

Combining together of an object’s properties and the methods [1] Restricts the programmer’s access to the object’s data // Hiding of data [1] Data values can only be read/written using the methods of the class [1]

[Total: 13] 5 (a) Last item added is the first item to leave // or equivalent wording [1]

R. LIFO (b) (i) HARRIS [1]

17843 [1] (ii) PROCEDURE PushJob

IF TopOfStack = 1000 [1] THEN

OUTPUT “Stack is already FULL”

ELSE

INPUT NewUserID

INPUT NewReferenceNo [1] TopOfStack ← TopOfStack + 1 [1] SpoolJob[TopOfStack].JobReference ← NewReferenceNo

SpoolJob[TopOfStack].UserID ← NewUserID [1] ENDIF

ENDPROCEDURE

M.Mushtaq Hussain

Contact the Teacher: 03215275281

Page 135: UNIVERSITY OF CAMBRIDGE INTERNATIONAL EXAMINATIONS …iteach.pk/wp-content/uploads/2015/02/computer-science-papers-A1-… · COMPUTING 9691/31 Paper 3 May/June 2011 2 hours ... 10

Page 7 Mark Scheme Syllabus Paper

GCE A LEVEL – October/November 2013 9691 31

© Cambridge International Examinations 2013

7 (a) Possible answers include: Encryption of email traffic [1] Email data if intercepted cannot be read [1] Encryption of passwords [1] Designed to prevent unauthorised access [1]

(b) Encryption algorithm ... The calculation/process/sequence of steps for converting the message text/data [1] Encryption key A number/parameter used by the encryption algorithm // e.g. the displacement shift for transposing characters [1]

(c) Asymmetric encryption ...

Private key is known only to the owner//Public key is known by both parties [1] Public and private keys are obtained from the purchase of a digital certificate // Keys are generated at the start of a secure (e.g. web or email) session [1] EITHER … Sender will use their own private key [1] Receiver decrypts using the sender’s public key [1] OR …. Sender uses the recipient’s public key [1] Receiver decrypts using their own private key [1] MAX 3

(d) Authorisation ...

Different permissions granted to different users [1] Restricted access to certain data files/directories/physical devices [1] User IDs MAX 1 Authentication Passwords [1] (Digital) signature // (Digital) certificate [1] Use of biometric data and methods [1] MAX 1

[Total: 11]

M.Mushtaq Hussain

Contact the Teacher: 03215275281

Page 136: UNIVERSITY OF CAMBRIDGE INTERNATIONAL EXAMINATIONS …iteach.pk/wp-content/uploads/2015/02/computer-science-papers-A1-… · COMPUTING 9691/31 Paper 3 May/June 2011 2 hours ... 10

CAMBRIDGE INTERNATIONAL EXAMINATIONS

GCE Advanced Level

MARK SCHEME for the October/November 2013 series

9691 COMPUTING

9691/32 Paper 3 (Written Paper), maximum raw mark 90

This mark scheme is published as an aid to teachers and candidates, to indicate the requirements of the examination. It shows the basis on which Examiners were instructed to award marks. It does not indicate the details of the discussions that took place at an Examiners’ meeting before marking began, which would have considered the acceptability of alternative answers.

Mark schemes should be read in conjunction with the question paper and the Principal Examiner Report for Teachers. Cambridge will not enter into discussions about these mark schemes. Cambridge is publishing the mark schemes for the October/November 2013 series for most IGCSE, GCE Advanced Level and Advanced Subsidiary Level components and some Ordinary Level components.

M.Mushtaq Hussain

Contact the Teacher: 03215275281

Page 137: UNIVERSITY OF CAMBRIDGE INTERNATIONAL EXAMINATIONS …iteach.pk/wp-content/uploads/2015/02/computer-science-papers-A1-… · COMPUTING 9691/31 Paper 3 May/June 2011 2 hours ... 10

Page 3 Mark Scheme Syllabus Paper

GCE A LEVEL – October/November 2013 9691 32

© Cambridge International Examinations 2013

(d) (i) (Yes) since there is a not a repeated group of attributes [1] (ii) (Yes) since there is only a single attribute primary key

// there are no partial dependencies // all non-key attr. are dependent on the primary key [1]

(iii) There are dependent non-key attributes //

SupplierName and/or SupplierTelNo are dependent on SupplierID [1]

(iv) PRODUCT(ProductID, ProductDescription, RetailPrice, SupplierID)

[1] SUPPLIER(SupplierID, SupplierName, SupplierTelNumber) [1]

If primary key not-indicated penalise once only (e) Avoids data duplication/avoids repeated data // reduces data redundancy [1]

Avoids data inconsistencies [1] Ensures data integrity [1] MAX 2

(f) SELECT CustomerID, OrderNo [1]

FROM ORDER [1] WHERE OrderDate = #15/01/2014# AND PaymentMethod = ’D’

(AND ISPaid = TRUE) [1]

Do not penalise imprecise syntax in the WHERE line

[Total: 19] 3 (a) Temporary storage location [1]

general purpose/special (purpose) [1] Inside the (micro)processor [1] MAX 2

(b) (i) 3C [1] (ii) 271 [1] (iii) Fewer digits used to represent any number // long string difficult to interpret [1]

Less likely to make a mistake when copying/converting a digit string [1] Easy to convert from binary to hex (vice versa) than binary to denary [1] MAX 1 R. Hex is easier to understand/write

M.Mushtaq Hussain

Contact the Teacher: 03215275281

Page 138: UNIVERSITY OF CAMBRIDGE INTERNATIONAL EXAMINATIONS …iteach.pk/wp-content/uploads/2015/02/computer-science-papers-A1-… · COMPUTING 9691/31 Paper 3 May/June 2011 2 hours ... 10

Page 4 Mark Scheme Syllabus Paper

GCE A LEVEL – October/November 2013 9691 32

© Cambridge International Examinations 2013

(c) (i) 2 bytes [1] (ii) The Program Counter contains 30 [1]

MAR ← [PC] // MAR given the contents of the PC [1]

PC ← [PC] + 1 // PC is incremented [1]

MDR ← [[MAR]] // The contents of the address in MAR is copied to MDR [1]

CIR ← [MDR] // The contents of MDR are copied to CIR [1]

OR … If the candidate’s answer uses the suggested instruction: The Program Counter contains 30 [1] PC contents are copied to MAR [1] PC contents are incremented to 31 [1] The contents of address 30 / 2150 is copied to MDR [1] MDR contents / 2150 is copied to CIR [1] MAX 5

(d)

ACC

IX

Memory

Address

204

1 0

13

(13)

13 / ft

2

14

27

27

3

22

49

49

4

[4] [Total: 15]

M.Mushtaq Hussain

Contact the Teacher: 03215275281

Page 139: UNIVERSITY OF CAMBRIDGE INTERNATIONAL EXAMINATIONS …iteach.pk/wp-content/uploads/2015/02/computer-science-papers-A1-… · COMPUTING 9691/31 Paper 3 May/June 2011 2 hours ... 10

Page 7 Mark Scheme Syllabus Paper

GCE A LEVEL – October/November 2013 9691 32

© Cambridge International Examinations 2013

6 (a)

–126 1 0 0 0 0 0 1 0

–5 1 1 1 1 1 0 1 1 +

0 1 1 1 1 1 0 1

1 1

Mark as follows ... –126 binary [1] –5 binary [1] Correct final pattern (f/t from their –126 and –5) [1] Answer is incorrect since outside range possible represented with single byte // answer overflows// final bit pattern is NOT –131 [1]

(b) (i) Mantissa: +13/16 [1]

Exponent: +3 [1] Number: +13/16 × 2+3 // evidence of shifting the mantissa three places [1]

6.5 [1] MAX 3

(ii) The mantissa starts with the digits 01 // the first two bits in the mantissa are different [1] (iii) More bits used for the mantissa will result in greater accuracy/precision [1]

More bits used for the exponent will result in larger range of numbers [1]

[Total: 10]

M.Mushtaq Hussain

Contact the Teacher: 03215275281

Page 140: UNIVERSITY OF CAMBRIDGE INTERNATIONAL EXAMINATIONS …iteach.pk/wp-content/uploads/2015/02/computer-science-papers-A1-… · COMPUTING 9691/31 Paper 3 May/June 2011 2 hours ... 10

Page 8 Mark Scheme Syllabus Paper

GCE A LEVEL – October/November 2013 9691 32

© Cambridge International Examinations 2013

7 Possible answers include: (a) Encryption of email traffic [1]

Email data if intercepted cannot be read [1] Encryption of passwords // logging-in to “something” [1] Designed to prevent unauthorised access [1] Hospital patient records [1] Will safeguard the privacy/confidentially of data [1]

(b) Plain text

The (message) text/data/ before encryption // unaltered text/original text [1] Cipher text The (message) text after encryption [1]

(c) Symmetric encryption

The plain text /data is encrypted using ... [1] An encryption key [1] Decryption is done using the same/ or by implication key …… [1] and a matching decryption algorithm/process [1] MAX 3

(d) Authorisation

Different permissions granted to different users [1] Restricted access to certain data files/directories/physical devices [1] User IDs [1] MAX 1 Authentication Passwords [1] (Digital) signature // (Digital) certificate [1] Use of biometric data and methods [1]

MAX 1

[Total: 11]

M.Mushtaq Hussain

Contact the Teacher: 03215275281

Page 141: UNIVERSITY OF CAMBRIDGE INTERNATIONAL EXAMINATIONS …iteach.pk/wp-content/uploads/2015/02/computer-science-papers-A1-… · COMPUTING 9691/31 Paper 3 May/June 2011 2 hours ... 10

CAMBRIDGE INTERNATIONAL EXAMINATIONS

GCE Advanced Level

MARK SCHEME for the October/November 2013 series

9691 COMPUTING

9691/33 Paper 3 (Written Paper), maximum raw mark 90

This mark scheme is published as an aid to teachers and candidates, to indicate the requirements of the examination. It shows the basis on which Examiners were instructed to award marks. It does not indicate the details of the discussions that took place at an Examiners’ meeting before marking began, which would have considered the acceptability of alternative answers.

Mark schemes should be read in conjunction with the question paper and the Principal Examiner Report for Teachers. Cambridge will not enter into discussions about these mark schemes. Cambridge is publishing the mark schemes for the October/November 2013 series for most IGCSE, GCE Advanced Level and Advanced Subsidiary Level components and some Ordinary Level components.

M.Mushtaq Hussain

Contact the Teacher: 03215275281

Page 142: UNIVERSITY OF CAMBRIDGE INTERNATIONAL EXAMINATIONS …iteach.pk/wp-content/uploads/2015/02/computer-science-papers-A1-… · COMPUTING 9691/31 Paper 3 May/June 2011 2 hours ... 10

Page 3 Mark Scheme Syllabus Paper

GCE A LEVEL – October/November 2013 9691 33

© Cambridge International Examinations 2013

(iv) CAR(CarRegistrationNo, CarMake, CarModel, HirePriceCode, DepotID)

[1] DEPOT(DepotID, DepotAddress, DepotManager ) [1]

If the primary key is no indicated, penalise once only

(e) avoids data duplication [1] avoids data inconsistencies [1]

(f) SELECT HireID, CustomerID [1]

FROM HIRE [1]

WHERE CustomerID = ’C674’ AND CarRegistration = ‘456431’ [1]

[Total: 19] 3 (a) Temporary storage location [1]

Inside the (micro)processor [1] (b) (i) 127 [1] (ii) 123 [1] (iii) less digits used to represent any number [1]

Less likely to make a mistake when copying/converting a digit string [1] Easy conversion between binary and hex (vice versa) than binary and denary [1] MAX 1

(c) (i) 2 bytes [1] (ii) MAR ← [PC] // MAR given the contents of the PC [1]

PC ← [PC] + 1 // PC is incremented [1]

MDR ← [[MAR]] // The contents of the address in MAR is copied to MDR [1]

CIR ← [MDR] // The contents of MDR are copied to CIR [1]

OR, if the candidate uses the suggested instruction …. MAR is given value 40 // PC contents of 40 are copied to MAR [1] 7324/The contents of address 40 is copied to the MDR [1] PC is incremented from 40 to 41 [1] 7324/contents of location 40 is copied to CIR [1] MAX 5

M.Mushtaq Hussain

Contact the Teacher: 03215275281

Page 143: UNIVERSITY OF CAMBRIDGE INTERNATIONAL EXAMINATIONS …iteach.pk/wp-content/uploads/2015/02/computer-science-papers-A1-… · COMPUTING 9691/31 Paper 3 May/June 2011 2 hours ... 10

Page 4 Mark Scheme Syllabus Paper

GCE A LEVEL – October/November 2013 9691 33

© Cambridge International Examinations 2013

(d)

Memory address

ACC 153 160

13

0

13

13

150

151

151

23

36

36

151

152

152

[4] [Total: 15]

M.Mushtaq Hussain

Contact the Teacher: 03215275281

Page 144: UNIVERSITY OF CAMBRIDGE INTERNATIONAL EXAMINATIONS …iteach.pk/wp-content/uploads/2015/02/computer-science-papers-A1-… · COMPUTING 9691/31 Paper 3 May/June 2011 2 hours ... 10

Page 6 Mark Scheme Syllabus Paper

GCE A LEVEL – October/November 2013 9691 33

© Cambridge International Examinations 2013

5 (a) BOOLEAN [1]

Flags when the book title is found [1] STRING (for SearchBook) [1] OPENFILE Book.txt for Output

INPUT SearchBook [1]

IsFound � FALSE

REPEAT

READ next book data value and assign to NextBook

IF NextBook = SearchBook [1] THEN

IsFound � TRUE OUTPUT “FOUND” ENDIF

UNTIL (IsFound = TRUE) OR EOF [1]

IF IsFound = FALSE // NOT IsFound [1] THEN OUTPUT “Book title was NOT FOUND” ENDIF

CLOSEFILE [1]

(b) The search will read on average 125 records [1]

(c) (i) The data items must be in order [1]

(ii) The function makes a call to itself (in two places) [1]

(iii) BinarySearch(BookTitle, “Tortoise Care”, 1, 11) High < Low is FALSE Middle = 6 BookTitle[6] > “Tortoise Care” is FALSE BookTitle[6] < “Tortoise Care” is TRUE

BinarySearch(BookTitle, “Tortoise Care” 7, 11) [1]

High < Low is FALSE

Middle = 9 [1] Booktitle[9] > “Tortoise Care” is FALSE Booktitle[9] < “Tortoise Care” is TRUE

BinarySearch(BookTitle, “Tortoise Care” 10, 11) [1]

High < Low is FALSE Middle = 10

BookTitle[10] > “Tortoise Care” is FALSE [1] Booktitle[10] < “Tortoise Care” is FALSE RETURN 10 ENDFUNCTION

ENDFUNCTION [1]

ENDFUNCTION

[Total: 16]

M.Mushtaq Hussain

Contact the Teacher: 03215275281

Page 145: UNIVERSITY OF CAMBRIDGE INTERNATIONAL EXAMINATIONS …iteach.pk/wp-content/uploads/2015/02/computer-science-papers-A1-… · COMPUTING 9691/31 Paper 3 May/June 2011 2 hours ... 10

Page 7 Mark Scheme Syllabus Paper

GCE A LEVEL – October/November 2013 9691 33

© Cambridge International Examinations 2013

6 (a) Boot file ... Stored in the BIOS/ROM [1] The initial sequence of instructions run when the computer is powered on [1] Information on which drive to look for the operating system [1] Triggers the loading of the operating system [1] MAX 3

(b) (i) An interrupt

a signal from some device [1] to indicate that some event has occurred [1] the device is seeking the attention of the processor [1] MAX 1

(ii) Hardware generated ...

reset [1] multiprogramming ‘end of time slice’ other valid answers ... Software generated ... [1] Division by zero error Other valid answers ... MAX 2

(c) RUNNING

The process currently has use of the processor [1] READY The process would like to use the processor when the current process releases the processor [1] SUSPENDED The process cannot currently use the processor// or by example, the job is currently using an I/O device [1]

[Total: 9]

M.Mushtaq Hussain

Contact the Teacher: 03215275281

Page 146: UNIVERSITY OF CAMBRIDGE INTERNATIONAL EXAMINATIONS …iteach.pk/wp-content/uploads/2015/02/computer-science-papers-A1-… · COMPUTING 9691/31 Paper 3 May/June 2011 2 hours ... 10

Page 8 Mark Scheme Syllabus Paper

GCE A LEVEL – October/November 2013 9691 33

© Cambridge International Examinations 2013

7 (a) (i)

Firewall

Hardware or software to control unauthorised access to a private network

Modem

Hardware used to convert analogue signals to digital signals (and vice versa)

Switch

Hardware used to connect nodes in a circuit switching network

Network Interface

card

Circuit board which connects the computer to a network

Router

Device to direct packets across a packet switched network

Bridge

Device used to connect two bus network segments to allow communication between all nodes

[5] (ii) Network (Interface) card [1] (b) (i) Copper wire/coaxial/twisted pair

Wire conducts electricity // changing current denotes different signals Optic fibre cabling Separate fibres used for separate signal Data travels very fast Signal transmitted as light pulses/travels at the speed of light Radio/Microwave signals Wireless communication // allows for mobile communication Mark as 2 × 2 MAX 4

(ii) Maximum possible distance [1]

Speed of communication // data transfer rate [1] MAX 1 [Total: 11]

M.Mushtaq Hussain

Contact the Teacher: 03215275281

Page 147: UNIVERSITY OF CAMBRIDGE INTERNATIONAL EXAMINATIONS …iteach.pk/wp-content/uploads/2015/02/computer-science-papers-A1-… · COMPUTING 9691/31 Paper 3 May/June 2011 2 hours ... 10

This document consists of 16 printed pages.

IB14 06_9691_32/2RP © UCLES 2014 [Turn over

*3948996442*

Cambridge International Examinations Cambridge International Advanced Level

COMPUTING 9691/32

Paper 3 May/June 2014

2 hours

Candidates answer on the Question Paper.

No additional materials are required.

No calculators allowed.

READ THESE INSTRUCTIONS FIRST

Write your Centre number, candidate number and name on all the work you hand in.

Write in dark blue or black pen.

You may use an HB pencil for any diagrams, graphs or rough working.

Do not use staples, paper clips, glue or correction fluid.

DO NOT WRITE IN ANY BARCODES.

Answer all questions.

No marks will be awarded for using brand names for software packages or hardware.

At the end of the examination, fasten all your work securely together.

The number of marks is given in brackets [ ] at the end of each question or part question.

M.Mushtaq Hussain

Contact the Teacher: 03215275281

Page 148: UNIVERSITY OF CAMBRIDGE INTERNATIONAL EXAMINATIONS …iteach.pk/wp-content/uploads/2015/02/computer-science-papers-A1-… · COMPUTING 9691/31 Paper 3 May/June 2011 2 hours ... 10

3

© UCLES 2014 9691/32/M/J/14 [Turn over

2 A company hires vehicles to customers. A database is to be created to record data for all hire transactions.

The company has a number of depots in different towns. Each town has one depot only. A number of vehicles are available for hire. Each vehicle is based at one of the depots. Every vehicle starts and ends a hire from its base depot.

Depot data consist of:

• Depot town name

• Depot address

Vehicle data consist of:

• A unique registration number

• A vehicle type code. Vehicles are coded as SC – Small Car, LC – Large Car or V – Van.

• Vehicle mileage

Customer data consist of:

• Unique Customer ID

• Customer address

Data for each hire transaction consist of:

• Customer ID

• Vehicle registration number

• Date the booking was made

• Start date of the hire

• Return date

• Driving licence check. A check is carried out to confirm that customers have a valid driving licence and this is recorded. This check is done when the customer shows their driving licence when they collect the vehicle.

• Hire charge

(a) At first, the company used a single table named DepotVehicle. A sample of the table’s

data is shown below.

DepotTown RegistrationNo VehicleType

Dhaka

0987 SC

0988 SC

0991 V

0945 V

Kumba

1431 SC

1476 LC

Explain why the table is not in normal form.

[2]

M.Mushtaq Hussain

Contact the Teacher: 03215275281

Page 149: UNIVERSITY OF CAMBRIDGE INTERNATIONAL EXAMINATIONS …iteach.pk/wp-content/uploads/2015/02/computer-science-papers-A1-… · COMPUTING 9691/31 Paper 3 May/June 2011 2 hours ... 10

4

© UCLES 2014 9691/32/M/J/14

The final design uses these four entities:

• Depot

• Vehicle

• Customer

• Hire.

(b) Consider the relationship between Depot and Vehicle.

(i) Draw the entity-relationship (E-R) diagram. [1] (ii) Complete the description for the Vehicle and Depot tables using the notation:

TableName(Attribute1, Attribute2, …)

where the underlined attribute indicates the primary key.

Vehicle( )

Depot( ) [4]

(c) The Customer and Hire tables (with the primary keys not shown) are as follows:

Customer(CustomerID, CustomerAddress) Hire(CustomerID, DateBooked, RegistrationNo, StartDate, ReturnDate, LicenceChecked, HireCharge)

Consider the relationship between Customer and Hire.

(i) Draw the E-R diagram.

[1] (ii) Explain how this relationship between Customer and Hire is modelled using the given

attributes.

[2]

M.Mushtaq Hussain

Contact the Teacher: 03215275281

Page 150: UNIVERSITY OF CAMBRIDGE INTERNATIONAL EXAMINATIONS …iteach.pk/wp-content/uploads/2015/02/computer-science-papers-A1-… · COMPUTING 9691/31 Paper 3 May/June 2011 2 hours ... 10

5

© UCLES 2014 9691/32/M/J/14 [Turn over

(d) The following Data Manipulation Language (DML) query is run: SELECT RegistrationNo FROM Hire WHERE (StartDate < Now() ) AND (ReturnDate > Now() );

Note: Now() is a function which returns the current date and time.

Describe what useful information is produced by the query.

[2]

(e) A query is needed to list the depot and registration number for all small car vehicles. Write the query in Data Manipulation Language (DML).

[3]

(f) On 05/04/2014, customer 085 booked a vehicle hire. This hire will start on 13/04/2014. When the customer arrives, the receptionist will check their driving licence. If the licence is

valid, the value of the licence check attribute, of the correct Hire record, must be updated.

Complete the Data Manipulation Language (DML) command to make the change.

UPDATE Hire

SET

WHERE CustomerID = '085' AND ; [2]

M.Mushtaq Hussain

Contact the Teacher: 03215275281

Page 151: UNIVERSITY OF CAMBRIDGE INTERNATIONAL EXAMINATIONS …iteach.pk/wp-content/uploads/2015/02/computer-science-papers-A1-… · COMPUTING 9691/31 Paper 3 May/June 2011 2 hours ... 10

11

© UCLES 2014 9691/32/M/J/14 [Turn over

5 (a) A computer system stores integers in 8-bit two’s complement form. Give the denary number represented by Byte 1 and Byte 2.

Byte 1 Byte 2

0 1 1 0 0 0 0 1 1 0 0 0 1 1 0 0

Byte 1

Byte 2 [2]

(b) Two bytes together are used to represent a Binary Coded Decimal (BCD) number. State the denary number represented by Byte 3 and Byte 4.

Byte 3 Byte 4

0 1 1 0 0 0 0 1 1 0 0 1 0 1 1 1

Denary: [2]

(c) Byte 5 and Byte 6 together represent a 16-bit colour code used in a drawing program.

Byte 5 Byte 6

0 1 1 0 1 0 1 0 1 1 1 1 0 1 0 1

The drawing program displays a colour code as a hexadecimal number.

State the hexadecimal number for this code. [2]

M.Mushtaq Hussain

Contact the Teacher: 03215275281

Page 152: UNIVERSITY OF CAMBRIDGE INTERNATIONAL EXAMINATIONS …iteach.pk/wp-content/uploads/2015/02/computer-science-papers-A1-… · COMPUTING 9691/31 Paper 3 May/June 2011 2 hours ... 10

12

© UCLES 2014 9691/32/M/J/14

(d) Many computer systems need to store and process real numbers. A computer uses two bytes to store a real number. The first (Byte 7) stores the mantissa and

the second (Byte 8), the exponent. Both mantissa and exponent use two’s complement. (i) What denary number is represented by Byte 7 and Byte 8?

Byte 7 Byte 8

0 1 1 0 1 0 0 0 0 0 0 0 0 0 1 1

Show your working.

[3]

(ii) Without any working out, how can you recognise that this 16-bit pattern (Byte 7 and Byte

8) is a positive number?

[1]

(e) (i) Without any working out, how can you recognise that this 16-bit pattern (Byte 7

and Byte 8) is normalised?

[1]

(ii) Both of the representations shown below are not normalised. Write in the empty rows the binary for the normalised form for the same value.

Mantissa Exponent

0 0 1 1 0 0 0 0 0 0 0 0 1 0 1 1

Mantissa Exponent

1 1 1 0 0 0 1 1 0 0 1 1 0 0 1 1

[3]

M.Mushtaq Hussain

Contact the Teacher: 03215275281

Page 153: UNIVERSITY OF CAMBRIDGE INTERNATIONAL EXAMINATIONS …iteach.pk/wp-content/uploads/2015/02/computer-science-papers-A1-… · COMPUTING 9691/31 Paper 3 May/June 2011 2 hours ... 10

13

© UCLES 2014 9691/32/M/J/14 [Turn over

(f) A change is made to use the two bytes as a 12-bit mantissa with a 4-bit exponent. Describe the effect of this change on the values that can be represented, compared with the old use of the two bytes.

[2]

M.Mushtaq Hussain

Contact the Teacher: 03215275281

Page 154: UNIVERSITY OF CAMBRIDGE INTERNATIONAL EXAMINATIONS …iteach.pk/wp-content/uploads/2015/02/computer-science-papers-A1-… · COMPUTING 9691/31 Paper 3 May/June 2011 2 hours ... 10

14

© UCLES 2014 9691/32/M/J/14

6 A business has a customer services section. The business is considering a new Local Area Network (LAN) for this section.

(a) Describe what is meant by a Local Area Network.

[2]

The following are some comments made by the Chief Executive to the IT Manager. (b) “I am concerned that only the staff in the Customer Services team should have access to the

LAN.” (i) State what authentication technique will be used for this.

[1]

(ii) Identify what hardware will be in place to restrict the access to the LAN.

[1]

(c) “I understand if the network is to be a wired network, there is a choice about the type of

cabling used.”

Name and describe two types of cabling. Suggest a benefit for each cable type. The benefits should be different.

Cable type 1

Description

Benefit

Cable type 2

Description

Benefit

[6]

(d) “Many customer enquiries will be dealt with over the World Wide Web; so each computer in

the LAN needs access to the Internet”. Name the additional hardware needed to provide access to the Internet.

[1]

M.Mushtaq Hussain

Contact the Teacher: 03215275281

Page 155: UNIVERSITY OF CAMBRIDGE INTERNATIONAL EXAMINATIONS …iteach.pk/wp-content/uploads/2015/02/computer-science-papers-A1-… · COMPUTING 9691/31 Paper 3 May/June 2011 2 hours ... 10

15

© UCLES 2014 9691/32/M/J/14 [Turn over

(e) “With the Internet connection, I am concerned that we will get unauthorised access to our LAN.”

Name the hardware and/or software needed to prevent unauthorised access.

[1]

(f) “Customer Services staff must be able to get access to a centralised store of customer data.

Our business deals with thousands of customers. They produce millions of transactions.” Explain what hardware and software will be needed to enable this.

Hardware

Software

[3]

M.Mushtaq Hussain

Contact the Teacher: 03215275281

Page 156: UNIVERSITY OF CAMBRIDGE INTERNATIONAL EXAMINATIONS …iteach.pk/wp-content/uploads/2015/02/computer-science-papers-A1-… · COMPUTING 9691/31 Paper 3 May/June 2011 2 hours ... 10

CAMBRIDGE INTERNATIONAL EXAMINATIONS

GCE Advanced Level

MARK SCHEME for the May/June 2014 series

9691 COMPUTING

9691/32 Paper 3 (Written Paper), maximum raw mark 90

This mark scheme is published as an aid to teachers and candidates, to indicate the requirements of the examination. It shows the basis on which Examiners were instructed to award marks. It does not indicate the details of the discussions that took place at an Examiners’ meeting before marking began, which would have considered the acceptability of alternative answers.

Mark schemes should be read in conjunction with the question paper and the Principal Examiner Report for Teachers. Cambridge will not enter into discussions about these mark schemes. Cambridge is publishing the mark schemes for the May/June 2014 series for most IGCSE, GCE Advanced Level and Advanced Subsidiary Level components and some Ordinary Level components.

M.Mushtaq Hussain

Contact the Teacher: 03215275281

Page 157: UNIVERSITY OF CAMBRIDGE INTERNATIONAL EXAMINATIONS …iteach.pk/wp-content/uploads/2015/02/computer-science-papers-A1-… · COMPUTING 9691/31 Paper 3 May/June 2011 2 hours ... 10

Page 3 Mark Scheme Syllabus Paper

GCE A LEVEL – May/June 2014 9691 32

© Cambridge International Examinations 2014

2 (a) The table is not in First Normal Form (1) The table has a repeated group of attributes / Registration and

VehicleType is repeated (for each depot) (1)

[2]

(b) (i)

[1]

(ii) Vehicle (RegistrationNo, VehicleType, Mileage,

DepotTown)

Mark as follows …

Correct first three attributes (1) RegistrationNo PK (1)

DepotTown present (1)

Depot (DepotTown (or similar), DepotAddress)

All correct … (1)

[4]

(c) (i)

Makes

[1]

(ii) Primary key CustomerID in the Customer table (1)

links to foreign key (CustomerID or by implication) in the Hire table (1)

[2]

(d) Displays the registration number (1) For all vehicles currently on a hireout (1)

[2]

(e) SELECT DepotTown, RegistrationNo (/ or equivalent) (1)

FROM Vehicle (R. ‘DepotVehicle’) (1)

WHERE VehicleType (/ or equivalent) = 'SC' (1)

[3]

(f) UPDATE Hire

SET LicenceChecked = TRUE // “YES” // equivalent (1)

A. Any sensible attribute name + value

WHERE CustomerID = '085' AND

StartDate = #13/07/2014#

// DateBooked = #05/04/2014# (1) [2]

M.Mushtaq Hussain

Contact the Teacher: 03215275281

Page 158: UNIVERSITY OF CAMBRIDGE INTERNATIONAL EXAMINATIONS …iteach.pk/wp-content/uploads/2015/02/computer-science-papers-A1-… · COMPUTING 9691/31 Paper 3 May/June 2011 2 hours ... 10

Page 7 Mark Scheme Syllabus Paper

GCE A LEVEL – May/June 2014 9691 32

© Cambridge International Examinations 2014

5 (a) 97 (1) –116 (1) [2]

(b) 61 97 1 mark per byte [2]

(c) 6A F5 1 mark per byte [2]

(d) (i) +6.5 give 3 marks If answer incorrect mark as follows: Exponent: +3 // move the pattern three places Mantissa: +13/16 // 0.1101

Answer: 13/16 × 23 // or equivalent [3]

(ii) (Positive …) The mantissa/byte 7 starts with a zero [1]

(e) (i) (Normalised …) The mantissa/byte 7 starts with 01 / the first two bits are different [1]

(ii) Mantissa Exponent

0 1 1 0 0 0 0 0 0 0 0 0 1 0 1 0

Mantissa Exponent

1 0 0 0 1 1 0 0 0 0 1 1 0 0 0 1

[MAX 3]

(f) The precision / accuracy is increased, but … The range of possible numbers is decreased [2]

M.Mushtaq Hussain

Contact the Teacher: 03215275281

Page 159: UNIVERSITY OF CAMBRIDGE INTERNATIONAL EXAMINATIONS …iteach.pk/wp-content/uploads/2015/02/computer-science-papers-A1-… · COMPUTING 9691/31 Paper 3 May/June 2011 2 hours ... 10

Page 8 Mark Scheme Syllabus Paper

GCE A LEVEL – May/June 2014 9691 32

© Cambridge International Examinations 2014

6 (a) A number of computers which are connected ... (1) Over a small ‘geographical’ area / or by example – site / building (1)

A. over a short distance [2]

(b) (i) Use of (user IDs with a) password Use of biometrics / fingerprint / retina scanner [1]

(ii) domain controller // file server to authenticate log-ons fingerprint/retina scanner

A. firewall [1]

(c) Possible benefits Optic fibre:

• data travels at the speed of light / provides for faster data transfer

• not affected by moisture / (electro-magnetic) interference

• impossible for the data to be hacked

• high bandwidth possible Twisted pair:

• less chance of interference from magnetic forces in close proximity

• low cost

• easy to work with // flexible Coaxial:

• difficult for the data to be hacked into

• screened to avoid (electro-magnetic) interference

Mark as: 2 × 3 marks per type [6]

(d) Router [1]

(e) Firewall //proxy server [1]

(f) Hardware … Large amount of hard-disk storage Database server Software … Database Management Software (A. DBMS) R. database Computers must have some form of ‘client software’ to access the database // Software must be specially written to access the DBMS // A. ‘browser’ if followed by some form of explanation [Max 3]

M.Mushtaq Hussain

Contact the Teacher: 03215275281

Page 160: UNIVERSITY OF CAMBRIDGE INTERNATIONAL EXAMINATIONS …iteach.pk/wp-content/uploads/2015/02/computer-science-papers-A1-… · COMPUTING 9691/31 Paper 3 May/June 2011 2 hours ... 10

This document consists of 18 printed pages and 2 blank pages.

IB14 06_9691_33/6RP © UCLES 2014 [Turn over

*6504753376*

Cambridge International Examinations Cambridge International Advanced Level

COMPUTING 9691/33

Paper 3 May/June 2014

2 hours

Candidates answer on the Question Paper.

No additional materials are required.

No calculators allowed.

READ THESE INSTRUCTIONS FIRST

Write your Centre number, candidate number and name on all the work you hand in.

Write in dark blue or black pen.

You may use an HB pencil for any diagrams, graphs or rough working.

Do not use staples, paper clips, glue or correction fluid.

DO NOT WRITE IN ANY BARCODES.

Answer all questions.

No marks will be awarded for using brand names for software packages or hardware.

At the end of the examination, fasten all your work securely together.

The number of marks is given in brackets [ ] at the end of each question or part question.

M.Mushtaq Hussain

Contact the Teacher: 03215275281

Page 161: UNIVERSITY OF CAMBRIDGE INTERNATIONAL EXAMINATIONS …iteach.pk/wp-content/uploads/2015/02/computer-science-papers-A1-… · COMPUTING 9691/31 Paper 3 May/June 2011 2 hours ... 10

3

© UCLES 2014 9691/33/M/J/14 [Turn over

2 Paintings by various artists are on display in art galleries all over the world. When a gallery holds an exhibition, it may display paintings:

• from its own stock, • or borrowed from other galleries, • or both.

A database is to be created. It will store data about artists, paintings, galleries and exhibitions. Artist data consist of:

• artist name (unique). Painting data consist of:

• a unique reference number (recognised by all galleries) • painting title • artist name • date (year only) • name of the gallery owning the painting.

Gallery data consist of:

• gallery name (unique) • country.

An exhibition only takes place once at one gallery and shows either:

• paintings for a single artist only, e.g. the Da Vinci exhibition at the National Gallery in London in 2011, or

• paintings by a number of artists.

Exhibition data consist of:

• exhibition title (unique) • gallery name • exhibition artist (if a single artist only, otherwise contains an empty string) • start date • closing date • all paintings which are included in the exhibition.

Painting-in-exhibition data consist of:

• exhibition title (unique) • painting reference number • loan fee.

When a gallery lends a painting to another gallery, it may charge a loan fee.

M.Mushtaq Hussain

Contact the Teacher: 03215275281

Page 162: UNIVERSITY OF CAMBRIDGE INTERNATIONAL EXAMINATIONS …iteach.pk/wp-content/uploads/2015/02/computer-science-papers-A1-… · COMPUTING 9691/31 Paper 3 May/June 2011 2 hours ... 10

4

© UCLES 2014 9691/33/M/J/14

The database design consists of five entities. These are shown in the entity-relationship (E-R) diagram.

ExhibitionGallery

ExhibitionPainting Artist

PaintingPaintsOwns

Features

(a) Consider the relationship between Gallery and Exhibition.

(i) Draw this relationship on the E-R diagram above. [1] (ii) Complete the description for the Gallery and Exhibition tables. Use the notation:

TableName(Attribute1, Attribute2, ...)

where the underlined attribute indicates the primary key.

Gallery ( )

Exhibition ( )[4]

(b) The Painting and ExhibitionPainting entities (with the primary keys not shown) are

as follows: Painting(PaintingRefNo, PaintingTitle, ArtistName, PaintingDate,

GalleryName)

ExhibitionPainting(ExhibitionTitle, PaintingRefNo, LoanFee)

(i) What is the primary key of table ExhibitionPainting?

[1]

Consider the relationship between Painting and ExhibitionPainting.

(ii) Draw this relationship on the E-R diagram. [1] (iii) Explain how this relationship is modelled using the given attributes.

[2]

M.Mushtaq Hussain

Contact the Teacher: 03215275281

Page 163: UNIVERSITY OF CAMBRIDGE INTERNATIONAL EXAMINATIONS …iteach.pk/wp-content/uploads/2015/02/computer-science-papers-A1-… · COMPUTING 9691/31 Paper 3 May/June 2011 2 hours ... 10

5

© UCLES 2014 9691/33/M/J/14 [Turn over

(c) Additional data are to be stored about each artist as follows: Artist(ArtistName, Nationality, YearBorn, YearDied)

The following Data Manipulation Language (DML) query is run. SELECT PaintingRefNo

FROM Painting, Artist

WHERE (Artist.ArtistName = Painting.ArtistName) AND

(YearBorn >= 1900) AND (YearDied <= 2000);

Identify what useful information is produced by the query.

[2]

(d) Write a DML query to display the painting reference number and painting title for all paintings

by Da Vinci.

[3]

(e) The painting ‘The Guitar Player’ was in the ‘Secrets and Silence’ exhibition. The

ExhibitionPainting table has a record for this. In the record:

• The painting is referred to by its reference number, 9065. • The loan fee is recorded as $10000. This is an error – there was no charge made for

the loan by the loaning gallery. Complete the DML command to amend this record.

UPDATE

SET

WHERE (ExhibitionTitle = 'Secrets and Silence')

AND ( ); [3]

M.Mushtaq Hussain

Contact the Teacher: 03215275281

Page 164: UNIVERSITY OF CAMBRIDGE INTERNATIONAL EXAMINATIONS …iteach.pk/wp-content/uploads/2015/02/computer-science-papers-A1-… · COMPUTING 9691/31 Paper 3 May/June 2011 2 hours ... 10

13

© UCLES 2014 9691/33/M/J/14 [Turn over

5 (a) A computer system stores integers as an 8-bit two’s complement integer. Give the denary number represented by Byte 1 and Byte 2.

Byte 1 Byte 2

1 0 1 0 0 0 0 1 0 1 1 0 1 1 1 1

Byte 1

Byte 2 [2]

(b) Two bytes together are used to represent a Binary Coded Decimal (BCD) number. State the denary number represented by Byte 3 and Byte 4.

Byte 3 Byte 4

0 1 0 0 0 0 0 0 0 1 0 1 0 1 1 1

Denary [2]

(c) Byte 5 and Byte 6 together represent a 16-bit colour code used in a drawing program.

Byte 5 Byte 6

0 1 1 0 1 1 1 0 1 1 1 1 1 0 0 1

The drawing program displays a colour code as a hexadecimal number.

State the hexadecimal number for this colour code. [2]

M.Mushtaq Hussain

Contact the Teacher: 03215275281

Page 165: UNIVERSITY OF CAMBRIDGE INTERNATIONAL EXAMINATIONS …iteach.pk/wp-content/uploads/2015/02/computer-science-papers-A1-… · COMPUTING 9691/31 Paper 3 May/June 2011 2 hours ... 10

14

© UCLES 2014 9691/33/M/J/14

(d) Many computer systems need to store and process real numbers. A computer uses two bytes to store a real number. The first (Byte 7) stores the mantissa and

the second (Byte 8) the exponent. Both mantissa and exponent use two’s complement. (i) What denary number is represented by Byte 7 and Byte 8?

Byte 7 Byte 8

1 0 0 1 0 0 0 0 0 0 0 0 0 0 0 1

Show your working.

[3]

(ii) How can you recognise that this 16-bit pattern (Byte 7 and Byte 8) is normalised?

[1]

(iii) The positive number 2.0 is to be represented as a normalised real number. Show the mantissa and exponent for this value.

Mantissa Exponent

[2] (iv) What is the largest positive number that can be represented? Use the same 8-bit

mantissa and 8-bit exponent. Show the mantissa and exponent.

Mantissa Exponent

Do not attempt to evaluate this. [2]

M.Mushtaq Hussain

Contact the Teacher: 03215275281

Page 166: UNIVERSITY OF CAMBRIDGE INTERNATIONAL EXAMINATIONS …iteach.pk/wp-content/uploads/2015/02/computer-science-papers-A1-… · COMPUTING 9691/31 Paper 3 May/June 2011 2 hours ... 10

15

© UCLES 2014 9691/33/M/J/14 [Turn over

(e) An alternative representation is suggested using a 6-bit mantissa with a 10-bit exponent. Describe the effect on the numbers which can be represented, compared to the 8-bit mantissa and 8-bit exponent used earlier.

[2]

M.Mushtaq Hussain

Contact the Teacher: 03215275281

Page 167: UNIVERSITY OF CAMBRIDGE INTERNATIONAL EXAMINATIONS …iteach.pk/wp-content/uploads/2015/02/computer-science-papers-A1-… · COMPUTING 9691/31 Paper 3 May/June 2011 2 hours ... 10

16

© UCLES 2014 9691/33/M/J/14

6 A firm of insurance brokers provides quotations. The firm uses two stand-alone computers. Some customers come in person to the office and are given a printed quotation to take away. Other customers enquire through the firm’s website over the Internet.

Each computer creates a file at the start of each day to log and save the quotations issued from

that computer. There has been a large increase in enquiries, so the firm is to employ three more staff to provide

quotations. The firm will design and implement a Local Area Network (LAN) of five computers. The LAN must provide all computers with access to the Internet.

(a) The network is to have a bus topology. Sketch the topology of the LAN. Clearly label all items of hardware. [5]

M.Mushtaq Hussain

Contact the Teacher: 03215275281

Page 168: UNIVERSITY OF CAMBRIDGE INTERNATIONAL EXAMINATIONS …iteach.pk/wp-content/uploads/2015/02/computer-science-papers-A1-… · COMPUTING 9691/31 Paper 3 May/June 2011 2 hours ... 10

17

© UCLES 2014 9691/33/M/J/14 [Turn over

(b) The LAN is implemented and the manager suggests that the firm should introduce an intranet.

(i) Describe what is meant by an intranet.

[2]

(ii) Describe four benefits that the provision of an intranet would bring to the business.

1

2

3

4

[4]

M.Mushtaq Hussain

Contact the Teacher: 03215275281

Page 169: UNIVERSITY OF CAMBRIDGE INTERNATIONAL EXAMINATIONS …iteach.pk/wp-content/uploads/2015/02/computer-science-papers-A1-… · COMPUTING 9691/31 Paper 3 May/June 2011 2 hours ... 10

18

© UCLES 2014 9691/33/M/J/14

(c) A quotation is accepted by a customer. The broker must ask the insurance company to confirm the quotation. A copy of the quotation is sent to the insurance company by encrypted email.

(i) Explain what is meant by symmetric encryption.

[2]

(ii) When the broker sends an encrypted email to the insurance company, asymmetric

encryption is used. Asymmetric encryption uses both private and public keys. To send encrypted emails to each other and receive them, Claude and Sobi each know

three keys.

CLAUDE SOBI

Knows:

CLAUDE’s Private Key

W

X

Knows:

SOBI’s Public Key

Y

Z

Four keys have the codes W, X, Y and Z as labels. For each of these keys, state who

owns it and whether it is private or public.

W

X

Y

Z [3]

M.Mushtaq Hussain

Contact the Teacher: 03215275281

Page 170: UNIVERSITY OF CAMBRIDGE INTERNATIONAL EXAMINATIONS …iteach.pk/wp-content/uploads/2015/02/computer-science-papers-A1-… · COMPUTING 9691/31 Paper 3 May/June 2011 2 hours ... 10

CAMBRIDGE INTERNATIONAL EXAMINATIONS

GCE Advanced Level

MARK SCHEME for the May/June 2014 series

9691 COMPUTING

9691/33 Paper 3 (Written Paper), maximum raw mark 90

This mark scheme is published as an aid to teachers and candidates, to indicate the requirements of the examination. It shows the basis on which Examiners were instructed to award marks. It does not indicate the details of the discussions that took place at an Examiners’ meeting before marking began, which would have considered the acceptability of alternative answers.

Mark schemes should be read in conjunction with the question paper and the Principal Examiner Report for Teachers. Cambridge will not enter into discussions about these mark schemes. Cambridge is publishing the mark schemes for the May/June 2014 series for most IGCSE, GCE Advanced Level and Advanced Subsidiary Level components and some Ordinary Level components.

M.Mushtaq Hussain

Contact the Teacher: 03215275281

Page 171: UNIVERSITY OF CAMBRIDGE INTERNATIONAL EXAMINATIONS …iteach.pk/wp-content/uploads/2015/02/computer-science-papers-A1-… · COMPUTING 9691/31 Paper 3 May/June 2011 2 hours ... 10

Page 2 Mark Scheme Syllabus Paper

GCE A LEVEL – May/June 2014 9691 33

© Cambridge International Examinations 2014

1 (a) (i) The rule is defined in terms of itself [1] (ii) Rule 4 [1] (b) (i) Valid – using rules 1 and 4 [1] (ii) Invalid – use of rule 4 [1] (iii) Invalid - <VarName> must start with a lower case character [1]

(iv) Valid – Use of all rules [3]

(c) <Underscore> ::= _

<ConstName> ::= <VarName><Underscore> [2]

2 (a) (i) [1]

(ii) Gallery(GalleryName, Country)

2 attributes (1) Primary key GalleryName (1)

Exhibition (ExhibitionTitle, GalleryName, ArtistName,

StartDate, CloseDate)

Correct five attributes (only) (1) Primary key ExhibitionTitle (1) [4]

(b) (i) ExhibitionTitle + PaintingReferenceNo [1]

(ii) [1]

(iii) Primary key PaintingReferenceNo in the Painting table (1)

Links to foreign key PaintingReferenceNo in the Exhibition-Painting

table (1) [2]

(c) Displays the Painting reference number (1) For all paintings by artists born after 1899 and died before 2001 (1) [2]

M.Mushtaq Hussain

Contact the Teacher: 03215275281

Page 172: UNIVERSITY OF CAMBRIDGE INTERNATIONAL EXAMINATIONS …iteach.pk/wp-content/uploads/2015/02/computer-science-papers-A1-… · COMPUTING 9691/31 Paper 3 May/June 2011 2 hours ... 10

Page 3 Mark Scheme Syllabus Paper

GCE A LEVEL – May/June 2014 9691 33

© Cambridge International Examinations 2014

(d) SELECT PaintingReferenceNo, PaintingTitle (1)

FROM Painting (1)

WHERE ArtistName = 'Da Vinci'; (1) [3]

(e) UPDATE ExhibitionPainting (1)

SET LoanFee = 0 (1) WHERE (ExhibitionTitle = 'Secrets and Silence')

AND PaintingRefNo = 9065; (1) [3]

3 (a) A class is the design / the blueprint (from which objects are later created) The class definition consists of properties and methods An object is an instance of a class An object must be based on a class definition Many objects can exist for the same class Properties of an object can only be accessed using methods provided (in the class definition) [MAX 3] (b) Properties and methods of a base class are available to a subclass A subclass can have properties and methods of its own. [MAX 2] (c) (i) Recognised notation for inheritance (1) EVENTVENUE class Address : STRING (1)

STADIUM + THEATRE class (and no other subclasses) (1)

STADIUM class SportsPlayed : STRING (1)

Declared as an ARRAY (1)

Capacity : INTEGER (1)

AllSeater : BOOLEAN (1)

TeamName : STRING (1)

Declared as an ARRAY (1)

THEATRE class NoOfSeats : INTEGER (1) EventType : STRING

Declared as an ARRAY (1)

Restarant : BOOLEAN (1)

[MAX 8]

Note: No mention of use of arrays scores MAX 7 (ii) Mark as follows: EVENTVENUE has Stadio Olympico + Rome (1) STADIUM has: Lazio + Roma + National rugby (1) Football and rugby (1) TRUE (all seater) (1) 80 000 (1) [5]

M.Mushtaq Hussain

Contact the Teacher: 03215275281

Page 173: UNIVERSITY OF CAMBRIDGE INTERNATIONAL EXAMINATIONS …iteach.pk/wp-content/uploads/2015/02/computer-science-papers-A1-… · COMPUTING 9691/31 Paper 3 May/June 2011 2 hours ... 10

Page 5 Mark Scheme Syllabus Paper

GCE A LEVEL – May/June 2014 9691 33

© Cambridge International Examinations 2014

(d)

SearchSport IsFound Current OUTPUT

LACROSSE FALSE 1

Moving right

3

Moving left

6

Found

TRUE

[MAX 5] One mark per entry (MAX 5) 5 (a) –95 [2] 111

(b) 4057 [2] (c) 6EF9 [2] (d) (i)–1.75 give 3 marks If answer incorrect mark as follows: Exponent: +1 // move the pattern one place Mantissa: –7/8

Answer: –7/8 × 21 // or equivalent [3] (ii) The mantissa starts with a 10 ... [1] (iii) 2.0 normalised … Mantissa Exponent

0 1 0 0 0 0 0 0 0 0 0 0 0 0 1 0

[2] (iv) Mantissa Exponent

0 1 1 1 1 1 1 1 0 1 1 1 1 1 1 1

Mantissa (1) Exponent (1) [2]

M.Mushtaq Hussain

Contact the Teacher: 03215275281

Page 174: UNIVERSITY OF CAMBRIDGE INTERNATIONAL EXAMINATIONS …iteach.pk/wp-content/uploads/2015/02/computer-science-papers-A1-… · COMPUTING 9691/31 Paper 3 May/June 2011 2 hours ... 10

Page 6 Mark Scheme Syllabus Paper

GCE A LEVEL – May/June 2014 9691 33

© Cambridge International Examinations 2014

(e) The precision/accuracy is decreased, but … The range of possible numbers is increased [2]

6 (a) Correct use of any of the following: Single segment of wire (1)

Terminators × 2 (1) Printer (1) File server (1) Firewall / Proxy server + Indication of a connection to the Internet (1) Router + Indication of a connection to the Internet (1) Modem + Indication of a connection to the Internet (1) [MAX 5] (b) (i) Intranet … Information system using Internet protocols Provides service of web pages (to client computers) // stores content on a web server System only available to staff within the organisation [MAX 2] (ii) Benefits of an Intranet Localised content available to relevant users only Uses standard protocols including HTTP/FTP and email protocols Good control over the accuracy of content (unlike the Internet) Provides centralised/single source of important company documents [4] (c) (i) Symmetric encryption … Plain text is changed Into a cipher text message using an encryption algorithm The same algorithm (and key) is used to decrypt the message [MAX 2] (ii)

Claude (W/X) Sobi (Y/Z)

Claude’s Public Sobi’s Private

Sobi’s Public Claude’s Public

Any correct × 3 [MAX 3]

M.Mushtaq Hussain

Contact the Teacher: 03215275281